You are on page 1of 128

ONE STOP DOC

G a s t r o i n t e s t i n a l
S y s t e m
One Stop Doc
Titles in the series include:
Cell and Molecular Biology Desikan Rangarajan & David Shaw
Editorial Advisor Barbara Moreland
Cardiovascular System Jonathan Aron
Editorial Advisor Jeremy Ward
Nervous System Elliott Smock
Editorial Advisor Clive Coen
Coming soon
Respiratory System Jo Dartnell and Michelle Ramsay
Editorial Advisor John Rees
Musculoskeletal System Bassel Zebian and Wayne Lam
Editorial Advisor Alistair Hunter
Renal and Urinary System and Electrolyte Balance Panos Stamoulos and Spyros Bakalis
Editorial Advisor Richard Naftalin and Alistair Hunter
Endocrine and Reproductive Systems Caroline Jewel and Alexandra Tillett
Editorial Advisor Stuart Milligan
Nutrition and Metabolism Miruna Canagaratnam and David Shaw
Editorial Advisor Barbara Moreland and Richard Naftalin
ONE STOP DOC
G a s t r o i n t e s t i n a l
S y s t e m
Miruna Canagaratnam BSc(Hons)
Fourth year medical student, Guys, Kings and
St Thomas Medical School, London, UK
Editorial Advisor: Richard J Naftalin MB CHB MSC PHD DSC
Professor of Epithelial Physiology, Kings College London Guys
Campus Centre for Vascular Biology and Medicine, London, UK
Series Editor: Elliott Smock BSc(Hons)
Fifth year medical student, Guys, Kings and
St Thomas Medical School, London, UK
A member of the Hodder Headline Group
LONDON
First published in Great Britain in 2004 by
Arnold, a member of the Hodder Headline Group,
338 Euston Road, London NW1 3BH
http://www.arnoldpublishers.com
Distributed in the United States of America by
Oxford University Press Inc.,
198 Madison Avenue, New York, NY10016
Oxford is a registered trademark of Oxford University Press
2004 Arnold
All rights reserved. No part of this publication may be reproduced or
transmitted in any form or by any means, electronically or mechanically,
including photocopying, recording or any information storage or retrieval
system, without either prior permission in writing from the publisher or a
licence permitting restricted copying. In the United Kingdom such licences
are issued by the Copyright Licensing Agency: 90 Tottenham Court Road,
London W1T 4LP.
Whilst the advice and information in this book are believed to be true and
accurate at the date of going to press, neither the authors nor the publisher
can accept any legal responsibility or liability for any errors or omissions
that may be made. In particular (but without limiting the generality of the
preceding disclaimer) every effort has been made to check drug dosages;
however it is still possible that errors have been missed. Furthermore,
dosage schedules are constantly being revised and new side-effects
recognized. For these reasons the reader is strongly urged to consult the
drug companies printed instructions before administering any of the drugs
recommended in this book.
British Library Cataloguing in Publication Data
A catalogue record for this book is available from the British Library
Library of Congress Cataloging-in-Publication Data
A catalog record for this book is available from the Library of Congress
ISBN 0 340 813431
1 2 3 4 5 6 7 8 9 10
Commissioning Editor: Georgina Bentliff
Project Editor: Heather Smith
Production Controller: Lindsay Smith
Cover Design: Amina Dudhia
Typeset in 10/12pt Adobe Garamond/Akzidenz GroteskBE by Servis Filmsetting Ltd, Manchester
Printed and bound in Spain
Hodder Headlines policy is to use papers that are natural, renewable and recyclable products
and made from wood grown in sustainable forests. The logging and manufacturing processes
are expected to conform to the environmental regulations of the country of origin.
What do you think about this book? Or any other Arnold title?
Please send your comments to feedback.arnold@hodder.co.uk
PREFACE vi
ABBREVIATIONS vii
SECTION 1 UPPER GASTROINTESTINAL TRACT 1
SECTION 2 LOWER GASTROINTESTINAL TRACT 35
SECTION 3 HEPATOBILIARY SYSTEM AND PANCREAS 73
INDEX 111
CONTENTS
PREFACE
From the Series Editor, Elliott Smock
Are you ready to face your looming exams? If you
have done loads of work, then congratulations; we
hope this opportunity to practice SAQs, EMQs,
MCQs and Problem-based Questions on every part
of the core curriculum will help you consolidate what
youve learnt and improve your exam technique. If
you dont feel ready, dont panic the One Stop Doc
series has all the answers you need to catch up and
pass.
There are only a limited number of questions an
examiner can throw at a beleaguered student and this
text can turn that to your advantage. By getting
straight into the heart of the core questions that come
up year after year and by giving you the model
answers you need this book will arm you with the
knowledge to succeed in your exams. Broken down
into logical sections, you can learn all the important
facts you need to pass without having to wade
through tons of different textbooks when you simply
dont have the time. All questions presented here are
core; those of the highest importance have been
highlighted to allow even sharper focus if time for
revision is running out. In addition, to allow you to
organize your revision efficiently, questions have been
grouped by topic, with answers supported by detailed
integrated explanations.
On behalf of all the One Stop Doc authors I wish
you the very best of luck in your exams and hope
these books serve you well!
From the Author, Miruna Canagaratnam
The gastrointestinal system is not boring, daunting,
or complicated, but can be fascinating, clear-cut and
painless to learn about and this book will you show
you how.
It is divided into three chapters based on the anatomy
of the gastrointestinal tract. Each chapter covers the
normal development, anatomy, physiology and phar-
macology of the system, focusing on the fundamen-
tals of what you will be tested on in exams. In addi-
tion it covers common pathology of the gut, for those
hoping to broaden their knowledge and impress their
tutors ahead of the clinical years!
Id like to extend my heartfelt thanks to Professor
Richard Naftalin, who has been an invaluable source
of information, constructive criticism and support.
Every fact and figure has been checked, double-
checked and then triple-checked by him. Without his
hard work, enthusiasm, and attention to detail it
would not have been possible to produce this book.
Id also like to thank Elliott for giving me the oppor-
tunity to do what I love best: write, and also for trust-
ing me to do the job.
I would like to dedicate this book to my mum and
dad.
ABBREVIATIONS
ACh acetylcholine
ALT alanine aminotransferase
APUD amine precursor uptake and
decarboxylation
AST aspartate aminotransferase
ATP adenosine triphosphate
cAMP cyclic adenosine monophosphate
CCK cholecystokinin
COX cyclooxygenase
DD duodenum
ECF extracellular fluid
GB gallbladder
GFR glomerular filtration rate
GGT gamma glutamyl transferase
GI gastrointestinal
GIP gastrointestinal inhibitory peptide
IP
3
inositol triphosphate
i.v. intravenous
IVC inferior vena cava
MALT mucosal-associated lymphoid tissue
NSAID non-steroidal anti-inflammatory drug
PGE
2
prostaglandin E
2
RBC red blood cell
sER smooth endoplasmic reticulum
This page intentionally left blank
UPPER
GASTROINTESTINAL TRACT
SECTION
1
SALIVA 2
THE ORAL CAVITY 4
SWALLOWING 6
DEVELOPMENT OF THE DIGESTIVE SYSTEM 8
THE STOMACH 10
PEPTIC ULCERS 12
GASTRIC ACID SECRETION 14
PEPTIC ULCERS SYMPTOMS AND
TREATMENT 16
GASTRIC SECRETIONS 18
THE OESOPHAGUS AND HIATUS HERNIA 20
SATIETY 22
PHYSIOLOGY OF THE GASTROINTESTINAL
TRACT 24
PHYSIOLOGY OF THE STOMACH 26
DISORDERS OF THE OESOPHAGUS AND
STOMACH 28
GUT MOTILITY 30
CONTROL OF GUT MOTILITY 32
UPPER
GASTROINTESTINAL TRACT
SECTION
1
1. Is it true or false that in the salivon (the functional subunit of the salivary gland)
a. Na
+
is passively reabsorbed in the excretory duct
b. The primary secretion is hypotonic to plasma
c. The composition of the saliva depends on its flow rate
d. The pH of saliva depends on its flow rate
e. Parasympathetic stimulation produces a thick mucoid secretion
2. Consider the salivon below
a. List the functions of salivation
b. Label the parts of the salivon indicated in the diagram below
3. Saliva secretion
a. Occurs only on stimulation of the sublingual gland
b. May increase by 15 times if food is present in the mouth
c. Can be stimulated by smell
d. Decreases at night
e. Is an unconditioned reflex
3
1
2
Upper gastrointestinal tract 3
EXPLANATION: SALIVA
Saliva has several functions in the oral cavity (2a):
It contains mucins, glycoproteins which provide lubrication to assist swallowing
It moistens mouth and tongue to facilitate speech
It contains IgA, the first defence against bacteria and viruses, as well as lysozyme, which attacks bacterial
walls
It initiates digestion: salivary alpha-amylase hydrolyses starch, lingual lipase breaks down triglycerides
It is alkaline to buffer mouth acid and maintain oral pH at 7.0, and also to prevent Na
+
ion loss.
Saliva is secreted by acinar cells, which make up units called salivons. On secretion it is isotonic to plasma. It
then undergoes secondary modification in the intercalated and excretory ducts. Na
+
is actively reabsorbed and
Cl

is passively taken up with it. K


+
and HCO
3

are actively secreted into the ducts. Saliva therefore reaches


the end of the duct, hypotonic to plasma (200 mosmol/L) and alkaline.
There are three pairs of salivary glands in the human.
Gland Type Secretion
Parotid Serous Watery
Submandibular Mixed Moderately viscous
Sublingual Mucous Viscous
The parotids and submandibular glands secrete on stimulation, but the sublingual gland secretes a thin
watery fluid all the time at a rate of about 0.5 mL/min. Around 1500 mL of saliva are secreted every 24 hours.
Saliva contains amylase, ribonuclease, R protein, lipase, lysozyme, IgA, IgG and IgM. Myoepithelial cells con-
tract around acini in response to stimuli to cause extrusion of fluid from zymogen granules inside the cells.
The composition of saliva varies with flow rate: increasing flow, increases salivary Na
+
and Cl

concentration,
because there is less time for ion reabsorption. At low flows, saliva is rich in K
+
but depleted of Na
+
and Cl

.
The presence of anything in the mouth (not necessarily food!) will stimulate saliva secretion. Parasympathetic
cholinergic stimulation produces watery secretion, and can be blocked by atropine and other anticholinergic
agents. Sympathetic adrenergic or noradrenergic stimulation produces mucus secretion.
Answers
1. F F T T F
2. a. See explanation. b. 1 acinus cell; 2 intercalated excretory duct; 3 myoepithelial cell
3. F T T T T
ONE STOP DOC 4
4. Concerning the anatomy of the oral cavity
a. It is divided into the vestibule and the mouth proper
b. The soft palate moves away from the wall of the pharynx during swallowing
c. All the muscles of the tongue are supplied by the hypoglossal nerve
d. General sensation to the anterior two-thirds of the tongue is supplied by the lingual
nerve
e. The tonsils lie posterior to both the palatoglossal and palatopharangeal arches
5. Regarding the tongue
a. Extrinsic muscles control its movement
b. It is attached to the floor of the mouth by the remnants of the thyroglossal duct
c. Vallate papillae lie anterior to sulcus terminalis
d. Section of the hypoglossal nerve causes deviation of the tongue towards the paralysed
side
e. The chorda tympani supplies taste to the anterior two-thirds of the tongue
6. Concerning the muscles of mastication
a. They are innervated by the fifth cranial nerve (trigeminal)
b. Buccinator is the principal muscle used in mastication
c. Masseter originates from the zygomatic arch
d. Lateral pterygoids act together to elevate the mandible
e. Medial pterygoids produce a grinding motion
Upper gastrointestinal tract 5
EXPLANATION: THE ORAL CAVITY
The oral cavity consists of the vestibule and the mouth proper. The roof is formed by the palate, and poste-
riorly the oral cavity communicates with the oropharynx. The oropharynx is bounded by the soft palate
above, the epiglottis below and the palatoglossal arches laterally.
Muscles of the soft palate arise from the base of the skull. They are levator veli palatine, tensor veli palatini,
palatoglossus, palatopharangeus and muscularis uvulae. The tonsils lie between the arches formed by
palatopharangeus and palatoglossus.
The tongue is divided in half by the lingual septum. In each half there are four extrinsic muscles and four
intrinsic muscles. The tongue is attached to the floor of the mouth by the lingual frenulum on its underside.
If you lift up your tongue the lingual veins running on either side of the frenulum can be seen.
All the muscles of the tongue except palatoglossus are innervated by the hypoglossal nerve (XII). Section of
the hypoglossal nerve results in paralysis and atrophy of one side of the tongue. The tongue deviates to the
paralysed side when it is stuck out because of the unopposed action of the unaffected genioglossus muscle on
the other side.
Movement of the temporal mandibular joint is mainly achieved by the muscles of mastication: temporalis,
masseter, medial pterygoid, which produce the biting motion, and lateral pterygoid which protudes the
mandible.
Answers
4. T F F T F
5. T F T T T
6. T F T F T
Epiglottis
Palatoglossal
arch
Circumvallate
papillae
Folate
papillae
Filiform
papillae
Fungiform
papillae
Midline
sulcus
Epiglottis
General sensory
and taste:
Internal laryngeal
nerve (X)
General sensory and taste:
Lingual branch of IX
General sensory
Lingual nerve (V)
Posterior 1/3
Anterior 2/3
Taste Chorda tympani (VII)
ONE STOP DOC 6
7. Arrange the following statements in chronological order
A. Elevation of the larynx
B. Relaxation of the upper oesophageal sphincter
C. Primary wave of peristalsis down the oesophagus
D. Mastication of food
E. Involuntary contraction of pharynx walls
F. Elevation of soft palate
G. Tongue raised against hard palate
8. True or false? Swallowing
a. Has four phases
b. Can be stopped once initiated
c. Is coordinated by the swallowing centre in the medulla
d. Is initiated by the tongue
e. Results in choking if the glottis is not closed
9. Regarding the pharynx, which of the following statements are true?
a. It is a common route for both food and air
b. It has three constrictor muscles
c. It constricts to prevent passage of food
d. The pharangeal plexus is formed from the branches of the eighth and ninth nerves
e. It is not involved in voice production
Upper gastrointestinal tract 7
EXPLANATION: SWALLOWING
The pharynx is a funnel-shaped tube of muscle that conducts food to the oesophagus and air to the larynx. It
is about 15 cm long and extends from the base of the skull to the inferior border of C6 vertebra. The pharynx
consists of three constrictor muscles: the superior, middle and inferior muscles, which overlap one another. It
also has three other muscles which descend from the styloid process (stylopharangeus), the cartilaginous part
of the auditory tube (salpingopharynegeus), and the soft palate (palatopharyngeus).
The three constrictors constrict the wall of the pharynx during swallowing, the other three muscles elevate
the pharynx and larynx during swallowing and speaking.
Swallowing has an initial voluntary phase followed by a second involuntary phase. In the first stage a deci-
sion is made to swallow. Food that has been chewed (masticated) is combined with saliva to form a bolus and
pushed to the oropharynx by the tongue. The tongue presses against the hard palate. The soft palate is elevated
to prevent food entering the nasopharynx.
In the second stage there is a reflex relaxation of the upper oesophageal sphincter and the walls of the pharynx
constrict to push the bolus down the pharynx into the oesophagus. The contraction of these muscles raises the
pharynx and the larynx. Both respiration and chewing stop during swallowing. The epiglottis reflexively
closes to prevent food entering the larynx.
Answers
7. 1 D, 2 G, 3 F, 4 B, 5 E, 6 A, 7 C
8. F F T T T
9. T T F F F
Superior constrictor muscle
Hyoid bone
Thyroid cartilage
Inferior constrictor muscle
Middle constrictor muscle
Cricoid cartilage
Oesophagus
Stylohyoid ligament
Right lateral view of pharynx
ONE STOP DOC 8
10. Concerning the development of the digestive system
a. The primitive gut tube forms in the fourth week
b. The mesenchyme gives rise to the epithelium of the gastrointestinal tract
c. The primitive gut is divided into three parts
d. The ventral part of the yolk sac is incorporated into the gut tube
e. Connective tissue and muscle are formed from the endoderm
11. Regarding development of the oesophagus
a. The oesophagus reaches its final relative length by week 7 of embryonic development
b. The oesophagus is separated from the laryngotracheal tube by the tracheoesophageal
septum
c. The glands of the oesophagus are derived from the endoderm
d. Smooth muscle develops from the splanchnic mesenchyme
e. Oesophageal atresia is caused by hypertrophy of the striated muscle from the branchial
arches
12. True or false? The stomach
a. Does not develop until the eighth week
b. Is attached to the liver by the dorsal mesentery
c. Grows faster dorsally than ventrally
d. Is continually orientated in the median plane
e. Is supplied from the coeliac trunk
GI, gastrointestinal
Upper gastrointestinal tract 9
EXPLANATION: DEVELOPMENT OF THE DIGESTIVE SYSTEM
The trilaminar embryonic disc that is formed in the third week of human development consists of three germ
cell layers: the ectoderm, mesoderm and endoderm. As the embryo develops, these layers give rise to the
tissues and organs of the embryo.
In the fourth week, folding of the embryo in median and horizontal planes converts the flat embryonic disc
into a C-shaped cylindrical embryo with a head and tail. The primitive gut tube emerges from the dorsal part
of the yolk sac with this sequence of folding events. It is divided into foregut, midgut and hindgut.
The endodermof the primitive gut tube gives rise to the epitheliumof the GI tract as well as the parenchyma
of the associated glands: the liver and pancreas. The splanchnic mesenchyme that surrounds the endoderm
gives rise to the connective tissue and muscles of the tract.
The oesophagus lengthens as a result of cranial body growth, and is usually at its final length by seven weeks.
The striated muscle that makes up the upper third of the oesophagus is formed from the mesenchyme of the
caudal branchial arches. The smooth muscle of the lower two-thirds of the oesophagus is derived from the
splanchnic mesenchyme.
Oesophageal atresia is a developmental abnormality that results from abnormal deviation of the tracheoe-
sophageal septum in a posterior direction, where there is incomplete separation of the laryngotracheal tube
from the oesophagus.
The stomach forms from a local dilation in the gut tube in the fourth week as shown in the above cross-section.
It is suspended from the dorsal wall of the abdominal cavity by a dorsal mesentery and attached to the liver
and ventral abdominal wall by a ventral mesentery.
Answers
10. T F T F F
11. T F T T F
12. F F T F T
Posterior abdominal wall
Dorsal mesentery
Ventral mesentery
Aorta
Right kidney
Gut
tube
ONE STOP DOC 10
13. On the diagram below of the developing stomach at six weeks, label the following
Options
A. Aorta
B. Dilation of foregut
C. Dorsal mesentery
D. Ventral mesentery
14. Concerning peritoneal coverings
a. The parietal peritoneum is held against the abdominal wall by the intra-abdominal fascia
b. The peritoneal cavity contains air only
c. Organs between the visceral and parietal peritoneum are termed retroperitoneal
d. The lesser sac lies behind the stomach
e. The greater omentum is fused to the transverse mesocolon
15. The following organs are retroperitoneal
a. Kidneys
b. Liver
c. Pancreas
d. Duodenum
e. Stomach
16. True or false? Pyloric stenosis
a. Affects 1 in 150 infants
b. Is caused by hypertrophy of the pylorus
c. Causes projectile vomiting
d. Can be felt as an enlarged mass in the right upper quadrant
e. Is untreatable
Posterior abdominal wall
90
Greater omentum
Upper gastrointestinal tract 11
EXPLANATION: THE STOMACH
The stomach tends to enlarge and broaden over the fifth and sixth weeks. It grows faster dorsally than ven-
trally, giving it its characteristic shape and
demarcating the greater curvature.
As shown below, the stomach rotates about
its longitudinal axis, 90 clockwise, so that
the left side faces anteriorly and the right
side posteriorly.
Since the mesenteries also rotate with the
stomach, there is formation of the greater
omentum by the dorsal mesentery, and the
omental bursa or lesser sac posterior to the
stomach. The lesser sac communicates with
the main part of the peritoneal cavity
through the epiploic foramen.
Congenital pyloric stenosis affects 1 in 150
male infants and 1 in 750 females. It is
caused by hypertrophy of the circular
muscles of the pylorus in the stomach. This narrow-
ing obstructs flow of food from the stomach to the
duodenum, and results in projectile vomiting,
without bile. The enlarged pylorus can be felt in the
right upper quadrant. It is usually treated by surgery.
Answers
13. See explanation
14. T F F T T
15. T F T T F
16. F T T T F
Posterior abdominal wall
Posterior
abdominal wall
90
Greater
omentum
Ventral
mesentery
Aorta
Greater
omentum
Dilation
of foregut
Dorsal
mesentery
Pancreas
Spleen
Left side of stomach is
now anterior
Greater omentum
Lesser
omentum
Aorta
Falciform ligament
Stomach
Greater curvature
Transverse
mesocolon
T. colon
Access to
lesser sac
via epiploic
foramen
ONE STOP DOC 12
17. Describe how the following contribute to the pathogenesis of peptic ulceration
a. Helicobacter pylori
b. Aspirin
18. Case study
A 46-year-old man is admitted to hospital with a 3-day history of sharp stomach pains and an
episode of vomiting blood. For the past two weeks he has been taking aspirin for a back injury.
On examination in the Emergency Department he was observed to be pale and tachycardic
with a low blood pressure. He was provisionally diagnosed with a gastric ulcer.
a. What may have caused the man to vomit blood?
b. How should he be treated?
c. How might this be prevented from happening again?
19. True or false? Helicobacter pylori
a. Is a gram-positive bacillus
b. Requires a pH lower than 3 for optimum growth
c. Is strongly associated with chronic gastritis
d. Secretes myeloperoxidase which destroys epithelial cells
e. May be eradicated by standard triple therapy
NSAID, non-steroidal anti-inflammatory drug; COX, cyclooxygenase; PGE
2
, prostaglandin E
2
; i.v., intravenous
Upper gastrointestinal tract 13
EXPLANATION: PEPTIC ULCERS
Helicobacter pylori is a gram-negative spiral-shaped bacteria which colonizes the gastric mucosa, most com-
monly in the antrum of the stomach. It is present in 70 per cent of patients with gastric peptic ulcers and
almost all patients with duodenal peptic ulcers. It is also present in 90 per cent of chronic active gastritis cases.
Antibiotic treatment of H. pylori promotes the healing of ulcers and prevents their recurrence.
It is thought that H. pylori impairs mucosal defenses by:
Secreting a protease that breaks down glycoproteins in gastric mucosa
Producing phospholipases which damage epithelial cells
Secreting a urease that generates free ammonia.
Neutrophils attracted to H. pylori release myeloperoxidase, which produces monochloramine in the presence
of ammonia. Monochloramine destroys cells (17a).
Due to intracellular urease activity the internal pH in H. pylori is buffered to neutral when the external pH
falls below 3.
Aspirin is an NSAID that causes irreversible inhibition of cyclooxygenase by acetylating a serine residue at
its active site. Therefore the production of prostaglandins by the COX pathway is inhibited. PGE
2
acts on pari-
etal cells to inhibit acid secretion into the gastric lumen. Removal of this inhibition encourages acid secre-
tion, which in turn causes gastric irritation, leading to a peptic ulcer (17b). People who take low doses of
aspirin for long periods to prevent myocardial infarction, unstable angina and stroke are at risk for peptic
ulcers.
Peptic ulceration is the erosion of a small solitary patch of the stomach or duodenum lining due to exposure
to acid peptic juices. The man in our case study is likely to have a peptic ulcer which has bled, causing him
to experience pain and vomit (18a). This man is suffering from hypovolaemic shock hence the need for
resuscitation and fluids. Treatment would involve (18b) resuscitation with i.v. saline or colloids, an H
2
antag-
onist (cimetidine) or proton pump inhibitor (omeprazole), a urea breath test for H. pylori, which can be
eradicated with triple therapy, stopping the aspirin and giving an alternative pain killer for back pain, and
finally lifestyle advice (reduce stress, stop smoking) (18c).
Answers
17. See explanation
18. See explanation
19. F F T F T
ONE STOP DOC 14
20. The following factors either (1) increase acid secretion or (2) impair mucosal defenses.
Label them accordingly
Options
A. Alcohol
B. Caffeine
C. Anticancer drugs
D. Stress
E. Hyperparathyroidism
F. Smoking
21. Describe briefly the mechanism of action of the following drugs used to inhibit gastric
acid secretion and give the class of each drug
a. Magnesium hydroxide
b. Ranitidine
c. Omeprazole
GI, gastrointestinal; NSAID, non-steroidal anti-inflammatory drug; cAMP, cyclic adenosine monophosphate
Upper gastrointestinal tract 15
EXPLANATION: GASTRIC ACID SECRETION
We become predisposed to conditions such as peptic ulceration, reflux oesophagitis and gastritis if there is
a chronic increase in acid secretion by parietal cells and/or an impairment in mucosal protection.
A peptic ulcer is the commonest cause of bleeding from the upper GI tract. It may be acute and severe
enough to cause hypovolaemic shock. Ingestion of NSAIDs, steroids or anticoagulants also predisposes to
bleeding.
Causes of increased acid secretion Causes of mucosal barrier breakdown
Caffeine stimulates cAMP activation Alcohol, vinegar and bile salts disrupt the unstirred
layer
Smoking NSAIDs, e.g. aspirin, disrupt the unstirred layer and
inhibit PG synthesis
ZollingerEllison syndrome characterized Anticancer drugs reduced regeneration of
by gastrin-secreting adenomas epithelial cells
Hyperparathyroidism raised plasma Ca levels H. pylori a gram-negative bacterium that colonizes
stimulate acid secretion gastric mucosa
Stress
Salts of magnesium/aluminium are commonly known as antacids. Taken orally they act to neutralize gastric
acid and raise gastric pH (21a). Ranitidine belongs to a group of drugs called histamine receptor antagonists.
It inhibits histamine-stimulated acid secretion by parietal cells in the gastric mucosa (21b).
Parietal cells have H
2
receptors on their cell surface membranes which when activated stimulate adenylate
cyclase to produce cAMP which in turn activates a protein kinase cascade, ending in stimulation of a proton
pump at the luminal surface of the cell. Inhibition of these receptors causes a decrease in the production of
cAMP, resulting in a decrease in the activity of the H
+
/K
+
ATPase (proton pump). Therefore less acid is
secreted into the stomach lumen.
Omeprazole is an irreversible proton pump inhibitor. This drug is activated at a pH lower than 3 to form a
disulphide link with the H
+
/K
+
ATPase, which directly blocks the pump, reducing H
+
ion secretion into the
lumen (21c).
Answers
20. 1 B, D, E, F; 2 A, C
21. See explanation
ONE STOP DOC 16
22. The following are symptoms of peptic ulceration. True or false?
a. Abdominal pain after eating b. Nausea
c. Vomiting of blood d. Water brash
e. Diarrhoea
23. Theme pharmacology of peptic ulcers. Match the following drugs with the
mechanisms of action listed below
Options
A. Omeprazole B. Cimetidine
C. Pirenzipine D. Magnesium hydroxide
E. Bismuth chelate F. Misoprostol
1. Analogue of prostaglandin E
1
to inhibit gastric acid secretion
2. Histamine receptor blockade
3. Coats gastric mucosa and eradicates H. pylori
4. Proton pump inhibition
5. Neutralizes gastric acid
6. Muscarinic receptor blockade on parietal cells
Upper gastrointestinal tract 17
EXPLANATION: PEPTIC ULCERS SYMPTOMS AND TREATMENT
Symptoms of peptic ulceration include:
Abdominal pain which gets worse at night but is relieved by food
Nausea and vomiting (may contain blood)
Poor sleep
In treating peptic ulceration, the general principles are to relieve pain, heal mucosa and prevent relapse. H.
pylori can be eliminated using triple therapy: two antibiotics and a proton pump inhibitor (e.g. amoxycillin,
metronidazole and omeprazole).
Antacids H
2
Histamine Proton pump Muscarinic
antagonists inhibitors receptor
antagonists
Taken Orally Orally Orally but enteric Orally
coated granules
in capsule
Action Neutralize gastric Block action of Block the H
+
/K
+
Block action of
acid, raise pH. Histamine on ATPase pump ACh on parietal
Inhibit peptic parietal cells. irreversibly. cells.
activity Reduce acid Reduce acid Reduce acid
secretion secretion secretion and
motility
Example Salts of Mg
2+
, Al
3+
Cimetidine, Omeprazole, Pirenzipine
ranitidine lansoprazole
Side Mg
2+
diarrhoea Rarely anti- Rare headache, Dry mouth,
effects Al
3+
androgenic, diarrhoea, rashes blurred vision
constipation gynaecomastia. (in 20% of
Acid rebound is Reduce hepatic patients)
a side effect of metabolism of
antacids other diagnoses
Effect on High doses Relieve pain of Particularly As effective as
ulcers promote ulcer ulcer and useful for severe cimetidine but
healing increase rate of ulceration side effects
ulcer healing resistant to other common
drugs
Answers
22. F T T T F
23. 1 F, 2 B, 3 E, 4 A, 5 D, 6 C
ONE STOP DOC 18
24. Sketch a graph to show the three phases of gastric acid secretion that follow feeding.
Indicate the stimulus necessary to initiate each phase
25. Concerning gastric secretions
a. The pylorus is the major secretory region in the stomach
b. Chief cells lie deep within the gastric glands
c. The acid provides optimum conditions for proteolytic enzyme activity
d. Intrinsic factor is not a necessary component of gastric secretion
e. The stomach only secretes gastric juice on stimulation
26. True or false? Gastrin
a. Is a 17 amino acid chain
b. Is secreted by parietal cells
c. Secretion is stimulated by caffeine
d. May be stimulated by local distension of the antrum
e. May be produced by certain tumours
ACh, acetylcholine; IP
3
, inositol triphosphate; cAMP, cyclic adenosine monophosphate; GIP, gastrointestinal inhibitory peptide; CCK, chole-
cystokinin
Upper gastrointestinal tract 19
EXPLANATION: GASTRIC SECRETIONS
On average, the gastric glands of the stomach produce 23 L of gastric juice per day, largely made up of
hydrochloric acid, pepsinogens and intrinsic factor. The fundus and the antrum are the main secretory
regions of the stomach.
Hydrochloric acid produced by the parietal cells, also called
oxyntic cells, has the following actions:
Kills many ingested bacteria
Lowers the pH to activate pepsin which starts protein digestion
Aids protein digestion
Stimulates flow of bile and pancreatic juice.
Acid is secreted from parietal cells (see diagram) via a proton
pump. Acid secretion by the cells is stimulated by three main
factors:
Gastrin (a peptide hormone)
ACh (a neurotransmitter)
Histamine (a local hormone).
The diagram below shows the three phases of gastric acid secretion
(24).
1. CEPHALIC PHASE Food in mouth reflexly stimulates
gastric acid secretion via efferent fibres of the vagus nerve.
ACh acts directly on parietal cells as well as G-cells in the
antrum and duodenum to release gastrin, and on mast-
like cells to release histamine.
2. GASTRIC PHASE Presence of food in the stomach
causes distension. This causes mechanoreceptors to initiate
a reflex cholinergic response. Amino acids or peptides act
directly on G-cells in the antrum to release gastrin.
3. INTESTINAL PHASE Presence of chyme in the duode-
num above pH 3 causes gastrin release. Presence of chyme
in the duodenum below pH 2 causes secretin release, which
inhibits gastrin secretion. Fatty acids cause release of GIP
and CCK, both of which depress acid secretion by parietal
cells.
Answers
24. See explanation
25. F T T F F
26. T F T T T
Parietal cell
Prostaglandin
Protein
kinases
Gastrin
Histamine
ACh
G-cells of antrum
and duodenum
mucosa
Mast-like cell
in lamina propria
Post ganglionic
neurons
H
+
K
+
Ca
2+
Ca
2+
Ip
3
AC
AMP cAMP
-
+
+
0 2 4
1
2
3
H
+
CONC
Time (hours)
ONE STOP DOC 20
27. True or false? Hiatus hernias
a. Are caused by the stomach riding up through the oesophageal hiatus
b. Result in regurgitation of food
c. May lead to a painful shoulder
d. Are associated with an increased risk of oesophageal carcinoma
e. Are common in young children
28. Case study
A 55-year-old man presents to the Emergency Department with a two-week history of
retrosternal chest pain, which comes on after eating, lying down or bending over. He also
complains of excess saliva in his mouth, and occasional difficulty swallowing. An ECG and X-
rays of the abdomen are ordered.
a. What is a hiatus hernia?
b. What investigation must be carried out to confirm the diagnosis?
c. How may this patient be treated?
29. True or false? The oesophagus
a. Lies posterior to the trachea in the superior mediastinum
b. Is compressed by the arch of the aorta
c. Pierces the diaphragm at the level of T12
d. Contains involuntary muscle in its upper third
e. Inclines to the right
30. Achalasia
a. Is caused by degeneration of the vagus nerve
b. Results in a chronically contracted lower oesophageal sphincter
c. Is virtually indistinguishable from Chagas disease
d. Causes dysphagia
e. Causes hyperpropulsive peristalsis
Upper gastrointestinal tract 21
EXPLANATION: THE OESOPHAGUS AND HIATUS HERNIA
The oesophagus is divided into cervical, thoracic and abdominal parts. It enters the superior mediastinum
between the trachea and vertebral column. Behind the trachea it inclines to the left so that it lies behind the
left bronchus in the posterior mediastinum. In its descent to the diaphragm, however, it crosses in front of the
descending aorta to the midline. It pierces the diaphragm at the level of T10. The muscle of the oesophagus
is voluntary in its upper third, and involuntary in its lower two-thirds.
Hiatus hernias may be sliding or paraoesophageal. Sliding occurs more commonly. They occur when intra-
abdominal pressure is higher than intrathoracic pressure; for example, when stooping, straining, coughing or
during pregnancy (28a). They may also occur if the clamping action of the right crus of the diaphragm on
the lower oesophagus is weak. Most people have no symptoms. However some experience reflux of food and
acid, which can lead to chronic irritation and ulceration. Other symptoms are water brash (sudden salivation)
and dysphagia. Referred pain to the shoulder may occur because the phrenic nerve innervates the parietal peri-
toneum and pleura as well as the pericardium next to the diaphragm. Pain is referred to the dermatomes of
C3, 4 and 5, the skin of the neck and shoulder.
A hiatus hernia may be demonstrated through a barium meal radiological examination (28b). Endoscopy
should also be performed to assess the presence of reflux oesophagitis and also to exclude carcinoma.
Symptoms may be managed non-operatively in 85 per cent of cases. Measures such as avoiding tight clothing
and bending, stopping smoking and losing weight can be beneficial. Surgery may be performed if symptoms
persist (28c).
Achalasia is obstruction at the lower oesophageal sphincter due to a failure of relaxation. It is thought to be
caused by degeneration of the ganglion cells in Auerbachs plexus. Chagas disease is an infection by
Trypanosoma cruzi which causes similar degenerative changes. The patient has difficulty swallowing (dyspha-
gia) since motility is disordered. Peristalsis is absent.
Answers
27. T T T T F
28. See explanation
29. T T F F F
30. F T T T F
1. SLIDING - Abdominal part of
oesophagus slides upwards
2. PARAOESOPHAGEAL - Cardia remains in place, part of the
stomach slips through the oesophages hiatus
Cardiooesophageal
junction
Diaphragm
Cardia
Stomach
Pouch of peritoneum
containing fundus
Diaphragm
Cardioesophageal
junction
Cardia
ONE STOP DOC 22
31. The following factors inhibit the feeding centre in the lateral hypothalamus in short-
term feeding regulation
a. Neuropeptide Y
b. Chewing
c. Insulin
d. Amino acids
e. Stomach distension
32. Regarding the control of food intake, are the following statements true or false?
a. Lesion to the ventromedial hypothalamus causes hyperphagia
b. Lesion to the lateral hypothalamus causes anorexia
c. Glucostats increase their activity to stimulate the feeding centre
d. Distension of the stomach is a satiety signal
e. Cholecystokinin is a satiety signal
33. Regarding the control of body weight
a. Leptin is secreted by enterocytes
b. Obese people have more fat cells than people of normal weight
c. Leptin counteracts the effects of neuropeptide Y
d. Obese people have lower leptin levels than people of normal weight
e. Excess leptin causes uncontrolled food intake
GI, gastrointestinal; CCK, cholecystokinin
Upper gastrointestinal tract 23
EXPLANATION: SATIETY
The feeling of fullness or satiety after eating results from stimulation of the satiety centre in the ventromedial
hypothalamus. Lesion to this centre causes hyperphagia. The feeding centre lies in the lateral hypothalamus.
Stimulation evokes feeding, whereas destruction causes anorexia. Glucose and amino acids inhibit the feeding
centre, as do chewing, swallowing and tasting. It is thought that normally the feeding centre is continuously
active. Its activity is inhibited transiently by the satiety centre after feeding.
Glucostats are central cells whose glucose utilization governs the activity of the feeding centre. When blood
glucose is low, the activity of these cells is decreased and therefore the activity of the feeding centre is
unchecked. When blood glucose is high, the utilization by these cells is high and therefore the activity of the
feeding centre is inhibited, thus the individual feels sated.
Satiety signals include stomach distension, which increases vagal activity via mechanoreceptors to inhibit
appetite, and CCK release, which (1) slows gastric emptying to maintain gastric distension (indirect effect) and
(2) reduces food intake by mimicking the vagal activity caused by gastric distension (direct effect). Smell, taste,
habit conditioning, chewing and swallowing cause satiety even before food reaches the stomach.
Leptin is a hormone secreted by adipocytes (fat cells). Circulating levels are proportional to the total amount
of fat in the body. Leptin acts at the hypothalamus to inhibit food intake. It counteracts the effects of neu-
ropeptide Y. Neuropeptide Y is released by the movement of foods along the GI tract and stimulates the
feeding centre. Leptin also increases fat metabolism, and increases energy expenditure.
Answers
31. F T T T F
32. T T F T F
33. F T T F F
ONE STOP DOC 24
34. Label the diagram of the GI tract below using the list provided
Options
A. Lamina propria B. Circular muscularis
C. Submucosa D. Lumen
E. Myenteric plexus
35. Theme physiology of the GI tract. Match the histological feature of the GI tract to its
function in the list below
Options
A. Parietal cell B. Chief (zymogen) cell
C. Mucous cell D. Brunners gland
E. Villi F. APUD cell
G. Goblet cell
1. Secretion of secretin and cholecystokinin
2. Pepsin secretion
3. Increased surface area for absorption
4. Secretion of acidic mucus in duodenal lumen
5. Acid secretion
6. Secretion of alkaline mucus in duodenal lumen
7. Secretion of mucus into stomach lumen
CCK, cholecystokinin; GI, gastrointestinal; APUD, amine precursor uptake and decarboxylation
Muscularis longitudinal
muscle (outer layer)
Muscularis
mucosa
Mucosa
Epithelium
1
3
5
4
2
Upper gastrointestinal tract 25
EXPLANATION: PHYSIOLOGY OF THE GASTROINTESTINAL TRACT
The GI tract is essentially a muscular tube lined by a mucous membrane. The major muscular component
remains relatively constant throughout the tract whereas the mucosa shows marked variations. The GI tract
has four distinct layers:
Mucosa
Submucosa
Muscularis: longitudinal and circular
Adventitia.
Within the mucosa, are three sublayers: the epithelium, lamina propria and muscularis mucosa.
Large clusters of parasympathetic ganglion cells are found between the two layers of the muscularis. This is the
myenteric plexus. Smaller clusters of parasympathetic ganglion cells in the submucosa that supply mucosal
glands and the smooth muscle of the muscularis mucosae are called Meissners plexus.
At different parts of the tract the mucosa changes from one form to another. The mucosa of the oral cavity,
pharynx, oesophagus and anal canal is protective, consisting of stratified squamous cells. Secretory mucosa in
the stomach consists of long, closely packed tubular glands that may be simple or branched.
The mucosa of the small intestine is entirely absorptive. The mucosa is arranged into finger-like projections
called villi, with short intervening crypts. The large intestine is lined with a combination of absorptive/pro-
tective mucosa. Tubular glands with cells specialized for water absorption and mucus-secreting goblet cells
are arranged closely packed together.
Answers
34. 1 D, 2 C, 3 A, 4 E, 5 B
35. 1 F, 2 B, 3 E, 4 G, 5 A, 6 D, 7 C
ONE STOP DOC 26
36. True or false? The stomach
A. Is where chyme formation takes place
B. Pylorus controls food entry to the stomach
C. Mucosa is thrown into folds called haustra
D. Has gastric pits which secrete gastric juice
E. Secretes gastrin from the pylorus
37. Regarding the stomach, which of the following statements are true?
a. It can hold up to 7 L of food
b. The fundus contacts the diaphragm
c. The cardiac sphincter is thickened by circular smooth muscle
d. Its blood supply is derived from the coeliac trunk
e. The stomach bed includes the pancreas
38. Consider the diagram of the stomach
a. What is indicated by the angular
notch?
b. Label the diagram of the stomach using
the following
Options
A. Body
B. Lesser curvature
C. Cardia
D. Oesophagus
39. Concerning the blood supply to the stomach
a. Three branches of the coeliac trunk supply the stomach
b. The left gastric artery supplies the greater curvature
c. The short gastric arteries branch from the splenic artery
d. The left gastroepiploic artery lies between the layers of the gastrosplenic ligament
e. The left gastric artery has an oesophageal branch
APUD, amine precursor uptake and decarboxylation
Fundus
Greater
curvature
Pyloric antrum
Angular notch
Pylorus
Duodenum
1
4
3
2
Upper gastrointestinal tract 27
EXPLANATION: PHYSIOLOGY OF THE STOMACH
The adult stomach can hold 23 L of food. It is fixed in place by two sphincters: the cardiac sphincter, which
is strengthened by the diaphragm curling around it to prevent regurgitation, and the pyloric sphincter, which
has circular smooth muscle to control flow of contents into the duodenum. The stomach bed upon which it
lies, consists of retroperitoneal structures: the pancreas, left kidney, suprarenal gland, transverse colon and the
spleen.
The mucosa of the stomach has three histologically different zones:
1. THE CARDIA Contains mostly mucus-secreting glands. They surround the entrance of the oesophagus.
2. THE FUNDUS AND BODY Contain gastric glands that consist of mucus-secreting mucus neck cells, pari-
etal cells which secrete acid, chief cells which secrete pepsinogen (a precursor of pepsin) and APUD cells. The
mucosa of the stomach is thrown into folds known as rugae which contain the gastric glands. They extend
from the muscularis mucosae to open into the stomach lumen via gastric pits.
3. THE PYLORUS Glands secrete mucus of two different types and have endocrine cells (G-cells) which
secrete the hormone gastrin.
The angular notch is two-thirds along the lesser curvature and marks the junction between the body of the
stomach and the pyloric part (38b). The left gastric artery arises from the coeliac trunk. The right gastric
artery arises from the hepatic artery. The right gastroepiploic artery arises from the terminal branches of the
gastroduodenal artery. The left gastroepiploic artery arises from the splenic artery. Short gastric arteries arise
from the splenic artery.
Answers
36. T F F F T
37. F T F T T
38a. See explanation
38b. 1 C, 2 B, 3 D, 4 A
39. T F T T T
Aorta
Left gastric
artery
Splenic artery
Short gastric
arteries
Spleen
Left gastroepiploic artery
Right gastroepiploic artery
Gastroduodenal artery
Hepatic artery
Coeliac trunk
ONE STOP DOC 28
40. True or false? Barretts oesophagus
a. Occurs because of acid reflux
b. Is the metaplastic transformation of columnar epithelium
c. Is more common in smokers
d. Should be monitored regularly
e. Increases the risk of squamous cell carcinoma
41. Regarding cancer of the stomach
a. Gastric adenocarcinoma is more common in women than men
b. Ingestion of smoked foods is a risk factor
c. Gastric adenomatous polyps predispose to adenocarcinoma
d. There are three different patterns of growth of the tumour
e. It presents very early
42. Gastric carcinoma
a. May follow gastritis
b. Is more common in those with blood group O
c. Is most likely to be an adenocarcinoma
d. May spread to supraclavicular lymph nodes
e. Is associated with increased gastric acid secretion
43. The following are risk factors for chronic gastritis. True or false?
a. Helicobacter pylori infection
b. Heavy consumption of alcohol
c. Trauma
d. Aspirin ingestion
e. Vegetarianism
NSAID, non-steroidal anti-inflammatory drug
Upper gastrointestinal tract 29
EXPLANATION: DISORDERS OF THE OESOPHAGUS AND STOMACH
Acid reflux oesophagitis may cause any of the following:
Peptic ulceration
Barretts oesophagus
Lower oesophageal stricture.
In Barretts oesophagus squamous epithelium of the lower oesophagus is replaced by glandular epithelium
composed of tall columnar cells (metaplasia). Patients should be kept under surveillance through endoscopy
and biopsy to detect early neoplastic changes. Barretts oesophagus puts the patient at risk of adenocarcinoma.
Gastritis is caused by inflammatory changes in the gastric mucosa and submucosa. It may be chronic or acute.
Chronic gastritis may be Helicobacter pylori associated, autoimmune associated with pernicious anaemia, or
reflux gastritis usually resulting from surgery to the pyloric region. This type of gastritis is also seen with pro-
longed NSAID use.
Complications of chronic gastritis include peptic ulceration and intestinal metaplasia, which may progress to
carcinoma.
Gastric adenocarcinoma is more common in men and is associated with blood group A and the ingestion of
smoked/salted preserved foods, due to the generation of nitrosamines by gut bacteria. People with adenoma-
tous polyps are particularly at risk because of malignant transformation, as are those with gastritis.
Answers
40. T F T T F
41. F T T T F
42. T F T T F
43. T T F T F
ONE STOP DOC 30
44. Regarding the enteric nervous system
a. Vasoactive intestinal polypeptide relaxes sphincters
b. The myenteric plexus lies in the submucosa
c. Meissners plexus lies between longitudinal and circular muscle
d. Stretch in the gut wall is sensed by neurons containing calcitonin-gene related
polypeptide
e. Plexuses are isolated from each other
45. Concerning GI smooth muscle
a. The basic electrical rhythm is set at three contractions per minute
b. The pacemaker lies on the lesser curvature
c. Smooth muscle is arranged in bundles
d. Most gastrointestinal smooth muscle undergoes tonic contraction
e. Slow waves are associated with Ca
2+
entry into the smooth muscle cells
GI, gastrointestinal; ACh, acetylcholine
Upper gastrointestinal tract 31
EXPLANATION: GUT MOTILITY
GI motility is important to move food, mix it and bring it into contact with absorptive cells. It is controlled
by both the extrinsic (autonomic) and intrinsic (enteric) nervous systems.
The smooth muscle of the gut is myogenic, that is it has its own spontaneous electrical activity. The pace-
maker region is on the greater curvature of the stomach. It sets the basic electrical rhythm at three waves per
minute. Impulses travel to the lesser curvature and the pyloric region. There are two types of electrical activity:
1. Slow waves are started in the pacemaker. They are oscillations within the resting membrane potential.
2. Spike waves occur at peaks of slow waves when membrane potential reaches a threshold of 40 mV.
Both types of wave cause Ca
2+
entry into the smooth muscle cells and therefore both types cause contraction.
However the type of wave determines the type of contraction. Most GI smooth muscle contracts and relaxes
in phases (phasic contraction) associated with slow waves and spikes. However sphincters undergo continuous
(tonic) contraction, as a result of high frequency spike potentials.
The enteric nervous systemconsists of two networks of fibres: the myenteric (Auerbachs) plexus and the sub-
mucous (Meissners plexus). They are interconnected within the GI tract. They contain secretory, motor and
sensory neurons. They secrete many neurotransmitters, most importantly ACh and noradrenaline. Peristaltic
activity is a result of the integrated function of the enteric system to sense the presence of food in the gut, and
contract and relax the appropriate muscle. Peristalsis generally gets stronger towards the antrum, so that gastric
contents are pressed towards the pylorus.
Answers
44. T F T T F
45. T F T F T
Slow waves
P
o
t
e
n
t
i
a
l

(
m
V
)
-40
-50
Pacemaker
Spike waves
Threshold
-40 mV
ONE STOP DOC 32
46. The following factors either (1) increase gastric motility, (2) decrease gastric motility,
or (3) do not affect gastric motility. Group them accordingly
A. Duodenal distension
B. Histamine
C. Gastrin
D. Parasympathetic nerve supply
E. Secretin
47. Concerning gastric motility, which of the following statements are true?
a. When food is swallowed the cardiac sphincter contracts
b. When food is swallowed the proximal stomach relaxes
c. Peristalsis is particularly strong in the antrum
d. Gastrin increases the frequency of the pacemaker
e. The pacemaker produces oscillations every 20 seconds
CCK, cholecystokinin
Upper gastrointestinal tract 33
EXPLANATION: CONTROL OF GUT MOTILITY
When food reaches the stomach, there is receptive relaxation of the lower oesophageal sphincter by the
vagus nerve.
Peristalsis is a reflex response controlled by the enteric nervous system but it may be increased or decreased
by the autonomic nervous system.
Parasympathetic cholinergic activity increases muscle contractions; sympathetic noradrenergic activity
decreases contractions but does contract the sphincters.
Distension of the duodenum by chyme causes a vagal reflex (iliogastric reflex), which controls the rate at
which it receives the chyme from the stomach.
Gastrin is released from G-cells in the stomach in response to changes in the gastric contents. Not only does
it stimulate gastric acid and pepsin secretion, it also increases gastric motility.
CCK is secreted in response to intestinal amino acids, it inhibits gastrin release and therefore inhibits gastric
emptying. It also augments the action of secretin, and stimulates satiety.
Secretin causes the contraction of the pyloric sphincter, reduces gastric motility, inhibits gastric secretion
and stimulates pancreatic secretion.
Answers
46. 1 C, D; 2 A, E; 3 B
47. F T T T T
This page intentionally left blank
LOWER
GASTROINTESTINAL TRACT
SECTION
2
MUSCLES OF THE ABDOMINAL WALL 36
THE INGUINAL REGION 38
INGUINAL HERNIAS 40
THE ANATOMY OF SMALL AND LARGE
INTESTINES 42
THE COLON 44
THE SPLEEN AND APPENDIX 46
DEVELOPMENT OF THE MIDGUT (i) 48
DEVELOPMENT OF THE MIDGUT (ii) 50
FUNCTION OF THE SMALL INTESTINE 52
ABSORPTION IN THE SMALL INTESTINE 54
ABSORPTION IN THE GUT 56
DISORDERS OF ABSORPTION 58
FLORA OF THE GUT 60
ABSORPTION IN THE COLON 62
CONSTIPATION AND LAXATIVES 64
BOWEL DISEASES 66
RECTUM AND ANUS 68
DEFECATION AND HAEMORRHOIDS 70
LOWER GASTROINTESTINAL
TRACT
SECTION
2
1. Concerning the muscles of the anterior abdominal wall
a. The rectus abdominis originates from the pubic symphysis and the pubic crest
b. Fibres of the external oblique run upwards and forwards
c. The rectus sheath is formed by the aponeurosis of three anterior wall muscles
d. The nerve supply to the wall is exclusively from the lower six intercostal nerves
e. The erector spinae muscles contract to allow you to sit up in bed
2. The rectus abdominis muscle
a. Flexes the trunk
b. Is covered anteriorly by the aponeurosis of the external oblique and internal oblique
c. Is divided into four smaller muscle segments
d. Has an arcuate line indicating where it is not covered by the sheath posteriorly
e. Is separated in the midline by the linea semilunaris
3. Concerning the posterior abdominal wall
a. The iliopsoas is a flexor of the trunk and an extensor of the thigh
b. Psoas major inserts in the greater trochanter of the femur
c. Quadratus lumborum lies lateral to psoas major
d. The lumbar plexus is composed of L1L3
e. The most superior nerve running across the wall is the subcostal nerve
Lower gastrointestinal tract 37
EXPLANATION: MUSCLES OF THE ABDOMINAL WALL
The external oblique muscle is most superficial. The aponeuroses of this muscle, the internal oblique
muscle and the innermost transversus abdominis, form the rectus sheath which encloses the rectus abdo-
minis a long muscle which lies on either side of the midline. External oblique fibres run downwards and
forwards; internal oblique fibres run upwards and outwards.
The rectus abdominis is a broad strap-like muscle extending from the pubic symphysis and the pubic crest,
attaching to the xiphoid process and the 5th7th costal cartilages. It flexes the trunk and thoracoabdominal
viscera. The muscle is enclosed by the rectus sheath which also contains superior and inferior epigastric arter-
ies and veins, lymphatic vessels and ventral primary rami of T7T12 nerves.
The posterior abdominal wall is made up from psoas major, iliacus and quadratus lumborum. The fibres of
the iliacus join the tendon of psoas. Together they form the chief flexor of the thigh. The ventral rami of L1L4
form the lumbar plexus which is in the posterior part of psoas major. The three largest branches of the plexus
are the obturator nerve, the femoral nerve and the lumbrosacral trunk.
Origin Insertion
Muscles of anterior abdominal wall
External oblique External surfaces of 5th Linea alba, pubic tubercle,
12th ribs anterior half of iliac crest
Internal oblique Thoracolumbar fascia, Inferior borders of
anterior two-thirds of iliac 10th12th ribs, linea alba,
crest, lateral half of inguinal conjoint tendon
ligament
Transverse abdominal Internal surfaces of 7th Linea alba with
12th costal cartilages, aponeurosis of internal
thoracolumbar fascia, lateral oblique, pubic crest,
third of inguinal ligament conjoint tendon
Rectus abdominis Pubic symphysis and pubic Xiphoid process, 5th7th
crest costal cartilages
Muscles of posterior abdominal wall
Psoas major Transverse processes of Lesser trochanter of femur
lumbar vertebrae, bodies of
T12L5 vertebrae
Iliacus Superior two-thirds of iliac Lesser trochanter of femur
fossa, ala of sacrum and and its shaft, psoas major
anterior sacroiliac ligaments tendon
Quadratus lumborum Medial half of inferior border Iliolumbar ligament, internal
of 12th rib and tips of lumbar lip of iliac crest
transverse processes
Answers
1. T F T F F
2. T T T T F
3. F F T F T
ONE STOP DOC 38
4. True or false? The inguinal ligament
a. Extends from the anterior superior iliac spine to the pubic crest
b. Is formed by the in-roll of the transverses abdominis aponeurotic fibres
c. Forms the pectineal ligament at the pubic crest
d. Contains the spermatic cord
e. Marks the location of the deep ring
5. The inguinal canal
a. Is 4 cm long in adults
b. Anterior wall is formed by the aponeurosis of the external oblique
c. Has a deep ring which lies just medial to the inferior epigastric artery
d. Has a non-palpable superficial ring
e. Is a rare site of hernias in both sexes
6. The spermatic cord
a. Passes through the inguinal canal
b. Has an innermost covering derived from the cremaster muscle
c. Carries the genital branch of the genitofemoral nerve
d. Drains lymph from the testis to the inguinal nodes
e. Can develop a hydrocele from the remnants of processus vaginalis
Lower gastrointestinal tract 39
EXPLANATION: THE INGUINAL REGION
The inguinal region is particularly important anatomically since it is an area of weakness in the abdominal
wall where hernias are prone to occur, especially in men. The inguinal ligament extends from the anterior
superior iliac spine to the pubic tubercle. It is formed by the in-roll of the aponeurotic fibres of the exter-
nal oblique. It forms the floor of the inguinal canal together with the lacunar ligament. The inguinal canal
contains the spermatic cord in males.
The spermatic cord is contained within three layers of fascia, derived from the anterior abdominal wall. They
are the internal spermatic fascia, derived from the transversalis fascia, the cremasteric fascia (and muscle)
from the fasica of the internal oblique muscle, and the external spermatic fascia derived from the external
oblique aponeurosis. The spermatic cord contains:
Three arteries testicular artery, cremasteric artery, artery of ductus deferens
Three nerves genital branch of genitofemoral nerve, ilioinguinal nerve fibres on arteries and ductus defer-
ens (sympathetic and parasympathetic)
Three veins testicular vein, cremasteric vein, pampiniform plexus
As well as the vas deferens and the lymphatics
Lymph from the testis drains to the para-aortic nodes, whereas lymph from the scrotum drains to the
inguinal nodes.
The inguinal canal is an oblique passage through the anterior abdominal wall. It has an opening at either end:
the deep and superficial rings. The deep ring lies lateral to the inferior epigastric artery, its pulsations are
a useful landmark to locate the ring.
Answers
4. F F F F F
5. T T F F F
6. T F T F T
Superficial
inguinal ring
Inguinal
ligament
Internal oblique
muscle
Reflected
external oblique
Conjoint tendon
Spermatic cord
Testis
External spermatic
fascia
Femoral vessels
ONE STOP DOC 40
7. Case study: inguinal hernia
A 52-year-old gardener comes into his GPs surgery one morning complaining of a lump in his
groin. He claims he first noticed it about a week ago when he had been trying to repair the
fence in his back garden. He felt a strange twinge as he lifted some planks of wood, but
thought it was nothing until he noticed the lump, which he says sometimes disappears when he
is lying down.
On examination, the lump was visible on the left side of the groin when the man stood. When
he lay down the lump disappeared. The GP placed two fingers over the deep ring and asked
the man to cough. When he removed his fingers the lump reappeared.
a. What is meant by the term indirect inguinal hernia? How does this differ from direct
hernia?
b. How might this man be treated?
c. What particular risks are involved with surgery to correct the hernia?
Lower gastrointestinal tract 41
EXPLANATION: INGUINAL HERNIAS
Sixty per cent of groin hernias are indirect inguinal. They are most likely to result from failure of obliteration
of the developmental processus vaginalis. The hernia protudes through the deep ring and along the inguinal
canal. It may descend into the scrotum. Indirect hernias may be reduced (stroked back) to the deep ring and
will not reappear unless you remove your finger from the deep ring, and raise intra-abdominal pressure (cough-
ing, standing).
Direct hernias are less common, and are caused by protusion of the bowel through the posterior abdominal
wall.
Uncomplicated hernias may be controlled using a truss (a pressure pad over the hernia) but it may be uncom-
fortable. Therefore, hernias may be repaired by open surgery or laparoscopic repair. In the case of indirect
hernias the hernial sac is excised and the deep ring is tightened. Direct hernias can be invaginated back with
sutures and reinforcement of the posterior wall of the inguinal canal.
Surgical excision endangers the genitofemoral nerve and ilioinguinal nerve (which provides cutaneous sen-
sation to the top of the thigh, scrotum and penis). Ductus deferens may also be affected. Recurrence after
surgery is common, usually due to incomplete excision of the sac. Also tightening the deep ring may constrict
the spermatic cord.
Answers
7. See explanation
ONE STOP DOC 42
8. The duodenum
a. Is entirely retroperitoneal
b. Relates closely to the head of the pancreas
c. Lies posterior to the inferior vena cava
d. Receives the pancreatic duct and bile duct in its second part
e. Lies between L1 and L3
9. True or false? The jejunum
a. Has closer packed plicae circulares than the ileum
b. Mesentery contains less fat than the ileum
c. Is less vascular than the ileum
d. Has more complex arterial arcades than the ileum
e. Derives its blood supply from the superior mesenteric artery
10. Regarding the large intestine
a. The longitudinal muscular coat consists of taeniae coli
b. The caecum is retroperitoneal
c. The transverse colon is mostly supplied by the inferior mesenteric artery
d. The inner wall of the colon contains Peyers patches
e. Its length can vary between 1.5 and 3.0 m
Lower gastrointestinal tract 43
EXPLANATION: THE ANATOMY OF SMALL AND LARGE INTESTINES
The small intestine connects the colon to the stomach. The duodenum is essentially the beginning of the small
intestine. It has four parts: superior, descending, horizontal and ascending. It is mostly retroperitoneal except
for the very first 2.5 cm. This part is mobile, and is known as the ampulla. The inferior vena cava, the aorta
and the right gonadal vein and artery lie behind the duodenum. Its blood supply is derived from the superior
mesenteric artery (proximally) and the coeliac trunk (distally).
The jejunum joins the duodenum at the duodenojejunal junction. It mostly lies in the left upper quadrant
of the abdomen. It is joined to the ileum together they form 67 m of the small intestine. They each have
distinctive characteristics, which are described in the table below. The mesentery is a fan of peritoneum that
attaches the jejunum and ileum to the posterior abdominal wall.
Jejunum Ileum
Calibre 3.0 cm 1.5 cm
Length 2.8 m 4.2 m
Vascularity Greater Less
Mucosa Circumferentially folded Randomly folded
plicae circulares
Mesentery Fewer arcades, large More arcades,
fenestrations small
fenestrations
Little fat More fat
Lymphoid nodules Peyers patches
The large intestine begins at the ileocaecal junction. Indeed, it consists of the caecum, appendix, and the
ascending, transverse and descending colon.
The caecum is a large dilated sac at the proximal end of the large intestine which lies in the right iliac fossa.
The ileum opens into the caecum at the ileocaecal valve. The large intestine is supplied by both the foregut
and midgut blood supplies. The distal third of the transverse colon onwards is supplied by the inferior mesen-
teric artery.
Answers
8. F T F T T
9. T T F F T
10. T T F F T
ONE STOP DOC 44
11. The large intestine can be distinguished from the small intestine by
a. The presence of taeniae coli
b. The fact that it is more vascular
c. Sacculations called haustra
d. Its larger internal diameter
e. Appendices epiploicae
12. Consider the cross-section of the colon below
a. Label the diagram of the ascending colon
appropriately, where indicated
b. What is the origin of the artery supplying this
part of the gastrointestinal tract?
c. Describe one way in which the large intestine
can be distinguished anatomically from the
small intestine
13. Regarding the GI tract
a. Label the parts indicated on the diagram
opposite
b. What is the function of (1)?
c. What is the blood supply of (2)?
GI, gastrointestinal
1
2
3
4
Lumen
3
1
2
Lower gastrointestinal tract 45
EXPLANATION: THE COLON
Taeniae coli are three flat bands that form the circular muscular coat of the large intestine. Because taeniae
are shorter than the intestine, the intestine is bunched up (sacculated) into haustra. There are no taeniae in
the appendix or rectum. Appendices epiploicae are small sacs of fat that hang off the large intestine.
The oesophagus passes through the diaphragm at the level of T10, where it meets the stomach at the cardiac
sphincter.
The cardiac and pyloric sphincters control the transit of food through the stomach. The pyloric sphincter is
thickened by smooth circular muscle, and the cardiac sphincter is supported by the right crus of the
diaphragm.
The developing GI tract is divided into foregut, midgut and hindgut, and the blood supplies of its compo-
nents are based on these divisions.
Foregut: coeliac trunk
Midgut: superior mesenteric artery
Hindgut: inferior mesenteric artery.
The first part of the duodenum lies at the level of L1. It is continuous with the stomach at the pylorus and is
part of the foregut.
The rest of the duodenum is retroperitoneal and is part of the midgut, therefore it derives its blood supply
from both the coeliac trunk (superior pancreaticoduodenal artery) and the superior mesenteric artery (infe-
rior pancreaticoduodenal artery).
The blood supply for most of the colon comes from the superior mesenteric artery (right and middle colic
branches) (12b). However the distal third and the descending colon are supplied by the left colic artery, a
branch of the inferior mesenteric artery. The inferior mesenteric artery continues inferiorly as the superior
rectal artery.
Answers
11. T F T T T
12a. 1. Appendices epiploicae; 2. Taeniae coli; 3. Diverticulum; 4. Circular muscle; 12b. See explanation; 12c. See explanation
13a. 1. Cardiac sphincter; 2. Pyloric sphincter; 3. Descending colon; 13b. To prevent regurgitation of food back up the oesophagus;
13c. Inferior mesenteric artery
ONE STOP DOC 46
14. True or false? The abdominal aorta
a. Enters the abdomen at T12
b. Bifurcates at L3
c. Bifurcates into the external and internal iliac arteries
d. Supplies the gut via three unpaired branches
e. Supplies the diaphragm via the superior phrenic arteries
15. The spleen
a. Is located in the right upper quadrant
b. Contacts the posterior wall of the stomach
c. Is retroperitoneal
d. Is supplied by the largest branch of the coeliac trunk
e. Is unlikely to be removed in the event of rupture
16. Regarding the appendix
a. It arises superior to the ileocaecal junction
b. It has a mesentery
c. It is supplied by a branch of the ileocolic artery
d. It is demarcated by McBurneys point
e. It is in the same position in everyone
Lower gastrointestinal tract 47
EXPLANATION: THE SPLEEN AND APPENDIX
The spleen is the largest lymphatic organ. It is located in the left upper abdominal quadrant, and is completely
covered in peritoneum except for the hilum. The relations of the spleen are: anteriorly, the posterior wall of
the stomach, posteriorly, the left part of the diaphragm, inferiorly, the left colic flexure and medially, the left
kidney. It is supplied by the splenic artery. The spleen may rupture on left-sided fracture to the 9th to 12th
ribs, or because of a sudden increase in intra-abdominal pressure. In this case, repair of the spleen is very dif-
ficult so a splenectomy (removal) is usually performed.
The vermiform appendix is a blind-ended pouch of intestinal diverticulum that arises just below the ileocae-
cal junction. It has a short mesentery mesoappendix by which it is attached to the caecum. The caecum is
supplied by the ileocaecal artery, a branch of the superior mesenteric artery. The appendix also derives its blood
supply from the ileocaecal artery. Usually the appendix is retrocaecal and extends up towards the right colic
flexure, however its position is variable between individuals. It may project towards the pelvis may even be
fused to the abdominal wall.
McBurneys point is one-third of the way along a line joining the right anterior superior iliac spine to the
umbilicus. The base of the appendix lies deep to this point.
Answers
14. T F F T F
15. F T F T F
16. F T T T F
Coeliac (foregut)
Superior mesenteric (midgut)
Inferior mesenteric (hindgut)
Renal
L4
Common iliac
External iliac
Internal iliac
Right testicular or ovarian
Aorta
T12
ONE STOP DOC 48
17. The following are derivatives of the midgut
a. Oesophagus
b. Rectum
c. Jejunum
d. Descending colon
e. Appendix
18. Regarding the development of the duodenum
a. It rotates with the stomach to the right
b. It lies externally to the peritoneum
c. It becomes partially occluded in the fifth and sixth weeks gestation
d. It has three different blood supplies
e. It is normally recanalized at birth
Lower gastrointestinal tract 49
EXPLANATION: DEVELOPMENT OF THE MIDGUT (i)
Derivatives of the midgut include: the small intestine (including much of the duodenum distal to the duo-
denal papilla), the caecum and appendix, the ascending colon, and most of the transverse colon (except the
distal third). The blood supply to the midgut is the superior mesenteric artery.
The duodenum epithelium arises from the caudal part of the foregut and the caudal part of the midgut. Other
parts of the duodenum arise from surrounding mesenchyme. The duodenum rotates to gain its characteristic
C-shaped loop, and becomes occluded in weeks 5 and 6 due to the proliferation of the epithelial lining cells.
The lumen is usually recanalized in the eighth week. Failure of recanalization results in duodenal
stenosis/atresia.
The entire midgut is suspended from the abdominal wall by an elongated dorsal (posterior) mesentery and
communicates with the yolk sac through the yolk stalk. As it lengthens, the midgut forms a ventral U-shaped
loop called the midgut loop. This projects into the proximal part of the umbilical cord because the liver and
primitive gut take up space in the abdominal cavity. This projection is often referred to as the midgut herni-
ation (19a). It takes place during the sixth week of development. It remains in the umbilical cord until the
end of the third month when it returns to the abdominal cavity (reduction).
The midgut herniation has cranial (headwards) and caudal (tailwards) parts: the cranial part lengthens rapidly
to form loops of the small intestine whereas the caudal part undergoes very little change except for the devel-
opment of a caecal bud, which later forms the caecum and appendix.
The midgut rotates 90 degrees anticlockwise around the superior mesenteric artery so that the cranial part is
brought to the right and the caudal part is brought to the left. When reduction takes place, the intestines turn
a further 180 degrees anticlockwise, so that the cranial part (the small intestine) is now superior to the caudal
part (19b).
Answers
17. F F T F T
18. T T T F F
ONE STOP DOC 50
19. Answer the following questions on the midgut development
a. What is meant by the term midgut herniation?
b. Describe briefly the rotation of the midgut
c. What is Meckels diverticulum?
20. Concerning the development of the anal canal
a. The anal canal has a dual origin
b. The cloacal membrane lies in the proctodeum
c. The urogenital sinus divides the cloaca
d. The ventral part of the cloaca develops into the anal canal
e. The superior two-thirds of the anal canal is supplied by the superior rectal artery
Lower gastrointestinal tract 51
EXPLANATION: DEVELOPMENT OF THE MIDUT (ii)
The final step in midgut development is fixation of the intestines. The mesentery attaching the intestines to
the posterior abdominal wall is modified once reduction has taken place. The mesentery of the ascending and
descending colon fuses with the parietal peritoneum on the posterior abdominal wall and disappears. As a
result they both become retroperitoneal. The colon presses the duodenum against the posterior abdominal wall
so that almost all of it becomes retroperitoneal.
Meckels diverticulum (ileal diverticulum) is the most common malformation of the gut. It is a remnant of
the proximal part of the yolk stalk that fails to degenerate during development, resulting in an outpouching
of the ileum. It is usually 5 cm long, and projects 50 cm from the ileocaecal junction. Two per cent of the pop-
ulation have Meckels diverticulum. It is usually symptomless but may cause rectal bleeding or abdominal pain.
The caudal part of the hindgut is called the cloaca. This cavity contacts the ectoderm at the cloaca membrane,
at the base of the proctodeum. The cloaca is divided into dorsal and ventral parts by the presence of the
urorectal septum. The ventral portion is called the urogenital sinus, whereas the dorsal portion is called the
anorectal canal. Where the urorectal septum fuses with the cloaca membrane is called the central perineal
tendon or perineal body. The epithelium of the superior two-thirds of the anal canal arises from the hindgut,
whereas the inferior third comes from the proctodeum. This dual origin is reflected in their differing blood
supplies. The superior two-thirds is supplied by the superior rectal artery from inferior mesenteric artery, the
inferior third is supplied by the inferior rectal arteries from the internal pudendal arteries.
Derivatives of the hindgut are: the distal third of the transverse colon, the descending colon, the sigmoid colon,
rectum, superior part of the anal canal, and the epithelium of the urinary bladder and most of the urethra.
Answers
19. See explanation
20. T T F F T
Superior
mesenteric
artery
Aorta
Liver
Stomach
Caudal limb
Midgut
loop
Cranial
limb
ONE STOP DOC 52
21. With regard to the secretions of the small intestine
a. Alkaline mucus is secreted by the Brunners gland
b. Bicarbonate ions are secreted in response to secretin
c. Enterocytes secrete water and electrolytes
d. Goblet cells secrete gastrointestinal hormones
e. The presence of chyme stimulates small intestine secretions
22. Fill in the gaps in the following paragraph about enzymatic aspects of digestion.
Use the words from the list provided
Options
A. Pepsin B. Brush border C. Secretin
D. Stomach lumen E. Oligopeptidase F. Acidic
G. Co-lipase H. Pancreas I. Emulsion droplets
J. Alkaline K. Salivary alpha amylase L. Acid lipases
M. Cholecystokinin N. Trypsin
Carbohydrate digestion begins in the oral cavity where starch is broken down by 1. Fats and
proteins are not broken down until they reach the 2. Here, amylase is inactivated by the 3 pH,
whereas 4 and 5 are activated. The mechanical action of the antrum is important to form small
6 of trigylcerides and fatty acids. Once chyme reaches the duodenum, 7 and 8 are released.
The 9 pH of the duodenum allows fatty acids to stabilize as smaller emulsion droplets. Pro-
enzymes from the 10 are activated by the 11 cascade. 12 is particularly important as it allows
pancreatic lipase to act on emulsion droplets. Protein digestion is completed by 13 on the
small intestine 14.
23. Concerning digestion in the small intestine, the following enzymes are present in the
lumen of the duodenum
a. Alpha-amylase
b. Pepsin
c. Lipase
d. 11-beta Hydroxylase
e. Esterase
GI, gastrointestinal; cAMP, cyclic adenosine monophosphate
Lower gastrointestinal tract 53
EXPLANATION: FUNCTION OF THE SMALL INTESTINE
The small intestine is responsible for digestion and absorption of end-products. On the surface of the small
intestine mucosa are small pits called crypts of Lieberkhn, lying between the intestinal villi. They, like the
villi, are covered by an epithelium composed of goblet cells, which secrete mucus, and enterocytes, which
secrete water and electrolytes. Around 1.8 L of intestinal fluid is secreted per day, predominantly by the ente-
rocytes. The secretions have a slightly alkaline pH. It should be noted that the watery secretion is reabsorbed
again by the villi thus it serves purely as a vehicle in which the products of digestion can be transported from
the chyme to the intestinal mucosal cells. The mucosa of the small intestine also contains solitary and aggre-
gated lymph nodules. Small glands in the duodenum called Brunners glands secrete alkaline mucus to protect
the duodenal mucosa from gastric acid.
Pancreatic secretions induced by the GI hormone secretin contain bicarbonate ions. Cl

is secreted into the


lumen via Cl

channels activated by cAMP. The digestive enzymes in the duodenum lumen are those secreted
from the pancreas (see page 107).
DIGESTION Starch is broken down by both salivary and pancreatic amylases, which break alpha 14 link-
ages between glucose residues. They do not act on 16 branch linkages. The disaccharides produced are broken
down in the small intestine lumen into monosaccharides by brush border enzymes such as sucrase, maltase
and lactase.
Fat digestion begins in the stomach lumen. Gastric lipase is produced in the fundus, and begins hydrolysis of
triglycerides. In the duodenum, bile secretion causes micelle/emulsion droplet formation which increases the
surface area for pancreatic lipases to act. They are secreted in pro-form, to be activated by trypsin. Co-lipase
displaces bile acids from emulsion droplets to allow lipase access to the triglycerides, which are broken down
to free fatty acids and glycerol.
Protein digestion also begins in the stomach lumen. Pepsin attacks peptide bonds at a low pH to release a few
amino acids and some smaller fragments of peptides. Once in the duodenum, endopeptidases activated by
trypsin continue the hydrolysis, assisted by brush border enzymes specific to certain amino acids.
Answers
21. T T T F T
22. 1 K, 2 D, 3 F, 4 L, 5 A, 6 I, 7 C, 8 M, 9 J, 10 H, 11 N, 12 G, 13 E, 14 B
23. T F T F T
ONE STOP DOC 54
24. Draw a diagram of a villus. Label the following on your diagram
A. Crypt of Lieberkhn
B. Lacteal
C. Microvilli
D. Muscularis mucosa
25. Regarding absorption in the intestine
a. Ninety per cent of Na
+
absorption takes place in the small intestine
b. K
+
is absorbed by active transport
c. Galactose and glucose compete for a co-transport carrier
d. Amino acid absorption is increased by the presence of Na
+
in the lumen
e. Free fatty acids are absorbed in the terminal ileum
ATP, adenosine triphosphate; ECF, extracellular fluid
Lower gastrointestinal tract 55
EXPLANATION: ABSORPTION IN THE SMALL INTESTINE
The absorptive surface of the small intestine is vastly increased in area by the valvulae conniventes (folds of
mucosa), villi (covering the mucosa) and microvilli (covering the villi). Villi are finger-like projections which
each contain a network of capillaries and a central lymphatic vessel called a lacteal. The epithelium of the villi
are further divided into microvilli which form the brush border which contains many of the important
enzymes of digestion.
ION ABSORPTION The enterocytes are permeable to Na
+
so it can enter the mucosa by diffusion. Their baso-
lateral membranes have Na
+
/K
+
ATPase pumps to maintain a favourable concentration gradient to absorp-
tion. Na
+
is also actively reabsorbed. Cl

ions tend to be dragged into the cells along with Na


+
. Water also
follows by osmosis. Some Na
+
is absorbed in exchange for secretion of K
+
or hydrogen ions. Hydrogen ions
secreted into the lumen combine with bicarbonate ions (from bile and pancreatic juice) to form carbonic acid
(HCO
3
). They then dissociate to form water and carbon dioxide. Carbon dioxide is then readily absorbed by
diffusion.
NUTRIENT ABSORPTION
Carbohydrate absorption of simple sugars such as glucose, fructose and galactose depends on specific
transport mechanisms (carrier proteins). Glucose transport depends on Na
+
concentration in the lumen
because glucose and Na
+
ions share the same transporter (symport). The Na
+
gradient required for the func-
tioning of this transport system is maintained by the basolateral Na
+
/K
+
ATPase pump. This is therefore an
active process (it requires ATP). Hexoses and pentoses are rapidly absorbed by simple diffusion across the
wall of the small intestine.
Protein L-amino acids are absorbed more rapidly than their D-isomers. D-Amino acids are absorbed almost
entirely by passive diffusion whereas most L-amino acids are actively transported. Transport is conducted by
a variety of carriers. It is thought that Na
+
and some amino acids are transported together in much the same
way that glucose is.
Answers
24. See diagram
25. T T T T T
Blood capillaries
Epithelial cell of virus
Submucosa
Microvilli
Central lacteal
Muscularis
mucosa
Crypt of
Lieberkuhn
GLUT 2
SGLT
Gut
lumen
ECF
Glucose Glucose
Na
+
2Na
+
ONE STOP DOC 56
26. Theme transport across the gut epithelium. Match the following nutrients to their
correct mechanism of absorption
Options
A. Simple passive diffusion
B. Facilitated diffusion
C. Primary active transport
D. Secondary active transport
1. Glucose
2. Alcohol
3. Fructose
4. Galactose
5. D-amino acids
6. Na
+
ions
7. Free fatty acids
27. Explain the following terms in association with the small intestine
a. MALT
b. Peyers patches
GI, gastrointestinal; MALT, mucosal-associated lymphoid tissue
Lower gastrointestinal tract 57
EXPLANATION: ABSORPTION IN THE GUT
Absorption of amino acids is rapid in the duodenum and jejunum but slower in the ileum. Basic, acidic and
neutral amino acids are transported by different mechanisms. A congenital defect in the mechanism for the
transport of neutral amino acids causes Hartnup disease, whereas a defect in the transport of basic amino acids
causes cystinuria.
Monoglycerides, cholesterol and fatty acids from micelles are absorbed by passive diffusion. Fatty acids
containing more than 1012 carbon atoms are esterified into triglycerides within the small intestine mucosal
cells. They are then packaged into chylomicrons and enter the lymphatics. Short-chain fatty acids pass
straight from the mucosal cells into the portal vein.
Water is absorbed entirely by diffusion along an osmotic gradient. It can be transported in both directions,
in other words from chyme to plasma, or plasma to chyme. This means that the chyme ends up isosmotic with
the plasma.
MALT stands for mucosal-associated lymphoid tissue. Lymphoid tissue is distributed throughout the GI tract,
either diffusely, or in discrete aggregations, for example the tonsils. The function of MALT is similar to that
of lymph nodes. Peyers patches are aggregations of lymphoid tissue in the small intestine, within the lamina
propria. They have germinal centres containing lymphocytes.
Answers
26. 1 D, 2 A, 3 B, 4 D, 5 A, 6 C, 7 A
27. See explanation
ONE STOP DOC 58
28. Divide the following causes of malabsorption into three groups: (1) very common,
(2) uncommon, (3) rare
A. Coeliac disease
B. Chronic pancreatitis
C. Whipples disease
D. Crohns disease
E. Resection of the stomach
29. The following are features of malabsorption. True or false?
a. Frothy greasy pale stools
b. Vomiting
c. Diarrhoea
d. Pernicious anaemia
e. Bleeding disorders
30. Coeliac disease
a. Is an abnormal reaction to starch
b. Is characterized by villous atrophy
c. Results in reduced gastrointestinal hormone secretion
d. Improves the prognosis for small bowel cancer
e. Is treated by cutting out carbohydrate
Lower gastrointestinal tract 59
EXPLANATION: DISORDERS OF ABSORPTION
Malabsorption in the small intestine occurs as a result of impairment of one of the following factors:
Secretion of pancreatic hydrolytic enzymes
Secretion of bile
Functional villi in the absorptive mucosa
Brush border enzymes.
Thus, pancreatic insufficiency is a common cause of malabsorption. Examples include cystic fibrosis,
chronic pancreatitis, cancer of the pancreas and pancreatic surgery. Coeliac disease is the most important
cause of malabsorption in industrialized countries.
Causes of malabsorption reducing the absorptive surface area of the mucosa include Crohns disease, infarc-
tion and resection of the small bowel.
The effects of malabsorption are:
Pale frothy stools which float because of their high fat and gas content (steatorrhea)
Osmotic diarrhoea: water remains in the lumen because of the inadequate absorption of osmotic particles
(nutrients)
Weight loss
Anaemia due to the deficiency in iron, folate or vitamin B12 caused by malabsorption
Bleeding disorders due to poor absorption of vitamin K (clotting factor)
Oedema resulting from low protein.
Coeliac disease is caused by an abnormal reaction to gluten found in wheat, rye, barley and oats. It causes
an enteropathy in the jejunal mucosa characterized by a loss of villous architecture and deepening of crypts.
The toxic element is the protein gliadin, a component of gluten. It induces a lymphocytic immune response.
Withdrawal of gluten from the diet leads to a recovery of normal architecture which may be partial or com-
plete. The condition may predispose to cancer of the small intestine.
Answers
28. 1 A, B; 2 D, E; 3 C
29. T F T T T
30. F T T F F
ONE STOP DOC 60
31. Match the following bacteria to the part of the alimentary tract where they are
commonly found
Options
A. Oropharynx
B. Stomach
C. Small intestine
D. Large intestine
1. Streptococcus viridans
2. Enterococcus faecalis
3. Clostridia
4. Helicobacter pylori
5. Lactobacillus
6. Escherichia coli
32. Concerning the flora of the intestinal tract
a. The colon is sterile at birth
b. Bacteria in the colon are predominantly aerobic
c. Bacteria convert conjugated bilirubin to urobilinogen
d. Bacteria convert fatty acids to simple sugars
e. Bacteria deconjugate sex hormones
33. Regarding antibiotic therapy
a. What type of antibiotic therapy is necessary when considering large bowel surgery?
b. Why might someone taking broad-spectrum antibiotics develop diarrhoea?
Lower gastrointestinal tract 61
EXPLANATION: FLORA OF THE GUT
The normal healthy human body has a whole array of commensal bacteria which exist symbiotically with
their host. This is known as the flora of the human body. These bacteria are harmless and may even be bene-
ficial in some cases.
In the gut, bacteria maintain the normal structure and function of the intestine. They degrade mucins,
epithelial cells and fibre, and they produce certain vitamins. They also convert conjugated bilirubin to uro-
bilinogen and stercobilinogen, and convert disaccharides to short-chain fatty acids. Commensal bacteria
usually help to prevent attack by pathogenic bacteria by competing for space and nutrients. However, com-
mensal bacteria can become pathogenic if they gain access to sites they do not usually inhabit. For this reason
it is vital to learn the normal distribution of flora. The main bacteria in the gut are listed below.
Site Organisms
Oropharynx Streptococcus viridans
Neisseria
Corynebacteria
Bacteroides
Stomach Usually sterile (H. pylori )
Small intestine Streptococcus
Lactobacillus
Colon Escherichia coli
Enterococci
Bacteroides
Proteus
Klebsiella
The stomach is usually sterile due to the secretion of gastric acid, but it may be colonized by Helicobacter
pylori. The flora of the small intestine is sparse, but the large intestine is very heavily colonized by anaerobic
bacteria. The bacteria in the faeces make up a third of its weight!
Before surgery on the intestines, the aim is to attain a site as clean as possible. Ideally cefuroxime is given to
kill gram-negative coliforms and neisseria, and metronidazole is given to kill gram-negative and -positive
anaerobes (33a). Antibiotics are generally given parenterally.
Diarrhoea is a common side effect of broad-spectrum oral antibiotic therapy. It occurs because the suppression
of normal commensal flora of the large intestine allows Clostridium difficile to overgrow (33b). C. difficile
is acquired from the environment or from other patients.
Answers
31. 1 D, 2 D, 3 B, 4 C, 5 D
32. T F T F T
33. See explanation
ONE STOP DOC 62
34. Concerning absorption in the colon
a. The colon can absorb a maximum of 5 L per day of fluid and electrolytes
b. Most of the absorption in the colon takes place in its proximal half
c. It is insensitive to aldosterone
d. Bicarbonate ions are absorbed in exchange for Cl

e. Paracellular spaces are important to the secretion of K


+
35. Diarrhoea
a. Is caused by a failure of water absorption in the intestines
b. Is never lethal
c. Causes a metabolic alkalosis
d. Can be caused by cholera toxin
e. May occur as a result of malabsorption of nutrients
ATP, adenosine triphosphate
Lower gastrointestinal tract 63
EXPLANATION: ABSORPTION IN THE COLON
One and a half litres of chyme pass into the large intestine each day. Ninety per cent of the water is absorbed,
leaving behind a small volume of faecal material which is mostly water. The mucosa of the large intestine has
a high capability for active absorption of Na
+
ions wherever Na
+
goes, water follows because of the osmotic
gradient created across the mucosa. Cl

ions also follow Na


+
. From the mucosal cell, Na
+
and Cl

are pumped
into paracellular spaces, in exchange for K
+
ions. K
+
is then driven into the lumen of the intestine because of
the electronegative potential in the lumen.
Tight junctions between the cells prevent back diffusion of Na
+
ions and aldosterone increases Na
+
transport.
The large intestine also secretes bicarbonate ions in exchange for the absorption of Cl

ions. Bicarbonate helps


to neutralize the contents of the lumen.
Diarrhoea may be osmotic, secretory, exudative or due to deranged motility. Basically, it occurs because the
mechanism by which the water is absorbed from the colon is impaired.
Diarrhoea causes the loss of water and bicarbonate ions (remember they are secreted into the lumen of the
intestine) from the body. The result is dehydration and a metabolic acidosis a surplus of hydrogen ions.
The person should take water and NaCl (i.e. oral rehydration solution) and allow the kidneys to restore the
normal balance of ions by excreting HCl and retaining NaHCO
3
.
Answers
34. T T F F T
35. T F F T T
Paracellular
space
Na
+
CI

HCO
3
Na
+
Na
+
H
+
K
+
H
2
O
H
2
O Osmosis
Osmosis
Tight junction
ATP ADP

ONE STOP DOC 64


36. Regarding laxative agents, which of the following statements are true?
a. Methylcellulose is a bulk laxative
b. They increase transit time through the colon
c. Lactulose has no side effects
d. Osmotic laxatives increase water content in the lumen
e. Magnesium hydroxide is a laxative
37. Constipation may be relieved by
a. Increasing the fibre content of the diet
b. Loperamide
c. Magnesium sulphate
d. Liquid paraffin
e. Stimulating the myenteric plexus
38. The following are important in the management of acute diarrhoea
a. Oral rehydration solution
b. Loperamide
c. Metoclopramide
d. Senna
e. Magnesium aluminium silicate
Lower gastrointestinal tract 65
EXPLANATION: CONSTIPATION AND LAXATIVES
There are two types of laxative:
1. BULK LAXATIVES act by increasing the volume of non-absorbable residue in the lumen of the intestine.
Examples are methycellulose, bran, agar. They take a few days to work, their advantage being that they have
no side effects, and are the most natural method of purging the colon.
2. OSMOTIC LAXATIVES act by increasing the volume of fluid in the lumen of the intestine by osmosis. This
reduces transit time through the small intestine, resulting in a large volume entering the large intestine, the
result is purgation within an hour. Examples are salts of Mg
2+
. They can cause abdominal cramps, diarrhoea
and electrolyte disturbances. Lactulose is an osmotic laxative.
Fibre adds bulk to stool to increase the mechanical efficiency of the intestine, and absorbs water to soften
the stool. Its deficiency is a very common cause of constipation in the elderly. Salts containing poorly absorbed
ions make good osmotic laxatives. Loperamide is an opioid used as an antidiarrhoeal not a laxative because of
its local effect.
Short episodes of diarrhoea are very common and often do not require clinical investigation or treatment.
However, if severe, causing dehydration or associated with fever, vomiting and abdominal pain, diarrhoea may
be a risk to the health of the vulnerable, particularly the old and the young.
Acute diarrhoea should be managed primarily with oral fluid and electrolyte replacement. Special rehydration
solutions such as diarolyte can be given. Although antidiarrhoeals are generally disapproved of because they
impair clearance of the pathogen, they can be given for short-term relief (e.g. loperamide). Adsorbent agents
such as charcoal, chalk and magnesium aluminium silicate have not been proved to provide any benefit,
but are thought to protect the intestinal mucosa.
Senna is a laxative and metoclopramide increases gut motility.
Answers
36. T F F T T
37. T F T T T
38. T T F F F
ONE STOP DOC 66
39. Regarding inflammatory bowel disease
a. Complete this table of differences between Crohns disease and ulcerative colitis
Crohns disease Ulcerative colitis
Location 1 Large intestine only, especially rectum
Inflammation Granulomatous pattern 3
Affects full thickness of bowel Affects mucosa and lamina propria
wall (transmural)
Discontinuous (skip lesions) Continuous
Ulceration 2 Shallow ulceration, haemorrhagic
Malignancy + risk 4
b. List three systemic complications of ulcerative colitis
40. True or false? Diverticular disease
a. Affects a third of the population over 35 years old
b. Is seen most commonly in the transverse colon
c. Is associated with raised intraluminal pressure
d. May result in perforation
e. May cause intestinal obstruction
41. Concerning appendicitis
a. Pain is initially felt in the umbilical region
b. Pain can be relieved by applying pressure to McBurneys point
c. It may cause peritonitis
d. It only affects children
e. It is treated by appendectomy
42. Hirschsprungs disease
a. Is caused by an absence of parasympathetic efferents to the gut
b. Is associated with children with Downs syndrome
c. Is often associated with early life gastroenteritis
d. May cause intestinal obstruction
e. Is characterized by megacolon
Lower gastrointestinal tract 67
EXPLANATION: BOWEL DISEASES
The two main chronic inflammatory bowel diseases are Crohns disease and ulcerative colitis. Inflammatory
bowel disease is generally more common in western countries. The cause is not known although there is a
familial tendency. Smoking increases the risk of Crohns but is protective against ulcerative colitis.
Symptoms depend on the region of the bowel affected, however both diseases tend to present as relapses and
remissions of diarrhoea, bleeding, abdominal pain and weight loss. Systemic manifestations include con-
junctivitis (eyes), arthritis, ankylosing spondylitis (joints), erythema nodosum (skin), fatty change in the liver
and chronic liver disease (39b).
Diverticular disease (diverticulosis) is the herniation of the mucosa and submucosa through the circular
muscle of the wall of the large intestine. It is rare under the age of 35 but affects a third of the population over
65 years old. Contributing factors are: weakness of the bowel wall, raised intraluminal pressure, an age-
related reduction in the strength of colonic connective tissue, and a low dietary fibre intake. Symptoms only
arise from complications, such as: inflammation resulting in diverticulitis, perforation of a pericolic abscess,
fistula formation, intestinal obstruction and bleeding.
Inflammation of the appendix causes early pain in the umbilical region because afferent pain fibres from the
appendix enter the spinal cord at T10. As the peritoneum overlying the appendix becomes inflamed, the pain
spreads to the right iliac fossa. Flexion of the right thigh relieves the pain somewhat, as it causes relaxation of
right psoas major.
Hirschsprungs disease affects 1 in 5000 babies. It is caused by the absence of ganglion cells from the
Auerbachs and Meissners plexus in the distal large bowel. It is normally associated with congenital abnor-
malities, particularly Downs syndrome.
Because the parasympathetic plexuses fail, peristalsis is lost and a segment of the bowel goes into spasm. This
causes the intestine proximal to the segment to become distended with faeces and dilate (megacolon).
Ischaemic enterocolitis may then develop, leading to death.
Answers
39a. 1. Anywhere in the alimentary tract, especially mouth, terminal ileum, colon, anus; 2. Deep ulcers and fissures, cobblestone appear-
ance; 3. No granulomas, goblet cell depletion, crypt abscesses; 4. +++ risk; 39b. See explanation
40. F F T T T
41. T F T F T
42. F T F T T
ONE STOP DOC 68
43. Regarding the rectum and anal canal
a. The lower third of the rectum is entirely covered by peritoneum
b. The anal canal begins as the bowel passes though the pelvic floor
c. The pectinate line demarcates a change in blood supply
d. The internal anal sphincter is supplied by voluntary nerve fibres
e. The internal anal sphincter contracts on sympathetic stimulation
44. Consider the rectum and anal canal
a. Explain briefly the mechanism of defecation
b. Label the diagram of the rectum and anal canal below
Options
A. Rectum
B. Sigmoid colon
C. Levator ani
1
2
3
Anal canal Bladder
Pubic symphysis
Uterus
Median section of the female pelvis
Lower gastrointestinal tract 69
EXPLANATION: RECTUM AND ANUS
The sigmoid colon and rectum are continuous at the level of vertebrae S2/S3. The rectum follows the curve
of the sacrum and the coccyx forming the sacral flexure. The upper third of the rectum is covered with peri-
toneum on its anterior and lateral surfaces, whereas the middle third is covered with peritoneum anteriorly
only. The inferior third is not covered at all. This is because the peritoneum is reflected forwards onto the
bladder in the male or vagina in the female.
On either side of the rectum are the levator ani muscles, or the pelvic diaphragm. The rectum mucous mem-
brane is thrown into transverse folds.
The superior rectal artery (from the inferior mesenteric artery) and the two middle rectal arteries supply the
proximal part of the rectum up to the pectinate line. Below the pectinate line (immediately below the anal
valves remnants of the embryonic proctodeum membrane), the inferior rectal arteries (from the internal
pudendal arteries) supply the anorectal junction and the anal canal.
The anal canal begins where the rectum pierces the pelvic diaphragm. It is about 2.53.5 cm long. The supe-
rior half of the anal canal has a mucous membrane characterized by folds called anal columns. The anorec-
tal junction is where the superior ends of the anal columns join the rectum; the inferior ends of the anal
columns form the anal valves.
The two sphincters controlling passage of faeces are concentric. The internal anal sphincter is smooth muscle.
It is basically a thickening of circular and longitudinal muscle from the rectum. The internal sphincter is ton-
ically contracted most of the time to prevent escape of faeces/fluid. It relaxes involuntarily in response to pres-
sure from faeces distending the rectal ampulla (44a). Part of the levator ani muscle, the pubo-rectalis forms a
sling around the ano-rectal junction, which assists the internal sphincter in contraction.
Skeletal muscle from the levator ani forms the external anal sphincter. This is a voluntary sphincter which is
supplied by the pudendal nerves (S4). It is voluntarily contracted to prevent defecation.
The anal canal ends at the anus the external outlet of the GI tract.
Answers
43. F T T T T
44a. See explanation; 44b. 1 B, 2 C, 3 A
ONE STOP DOC 70
45. Case study: Haemorrhoids
A 32-year-old woman comes to see her GP one afternoon because she has noticed blood
stains on the toilet paper after passing stool for the past 4 days. She is 26 six weeks pregnant
and worried something might be wrong with her baby. On further questioning, she admits that
she has noticed some mucus discharge in her stool.
a. What are haemorrhoids? How are they classified?
b. What are the risk factors for haemorrhoids?
c. Why might this woman feel tired and lethargic?
d. Is it possible her symptoms could be explained by any other pathology?
Lower gastrointestinal tract 71
EXPLANATION: DEFECATION AND HAEMORRHOIDS
Mass movement of faeces from the sigmoid colon to the rectum distends this chamber and causes stretch
receptors to send impulses via the parasympathetic afferents of sacral nerves 2, 3 and 4. In children who have
not yet been potty-trained, a reflex arc causes inhibition of the internal sphincter by sympathetic efferents from
L1 and 2 so that it relaxes, and there is also relaxation of the external sphincter via the pudendal nerves (S3
and 4). Presynaptic parasympathetic outflow causes the smooth muscle of the rectal wall to contract, resulting
in peristaltic waves to expel faeces. In older children and adults, ascending pathways from the spinal cord to
the cerebral cortex make us aware of the need to defecate, and therefore descending pathways inhibit the reflex
arc if inappropriate.
Haemorrhoids or piles, as they are more commonly known, are the commonest cause of rectal bleeding. The
anus is lined by three vascular cushions, attached by smooth muscle and elastic tissue. They drain via the
superior rectal veins into the inferior mesenteric vein. Gravity, increased anal tone and straining at stool all
weaken the supporting framework of these cushions, so they become loose and protude. They are then vul-
nerable to trauma and bleeding. Risk factors are a history of constipation, pregnancy, portal hypertension
and rectal carcinoma (45b).
Haemorrhoids are classified as primary, secondary and tertiary (45a). Primary piles remain in the rectum.
Secondary piles prolapse through the anus on defecation but spontaneously reduce. Tertiary piles remain
outside the anus at all times. Bright red rectal bleeding, mucus discharge, pruritus ani and anaemia are all
features (45c).
It is possible that in this case study the womans symptoms are caused by a perianal haematoma, anal fissure
or abscess, or even a tumour (45d). Therefore further investigations are always required.
Answers
45. See explanation
This page intentionally left blank
HEPATOBILIARY SYSTEM
AND PANCREAS
SECTION
3
THE LIVER STRUCTURE AND FUNCTION 74
THE LIVER ANATOMY 76
LIVER MICROSTRUCTURE 78
BILE SECRETION 80
THE GALLBLADDER 82
GALLSTONES 84
THE INSULIN RECEPTOR AND THE EFFECTS
OF INSULIN 86
DEVELOPMENT OF THE LIVER AND
PANCREAS 88
INSULIN 90
LIVER FAILURE 92
HEPATIC PATHOLOGY 94
CIRRHOSIS OF THE LIVER 96
THE PANCREAS 98
JAUNDICE AND DRUG METABOLISM 100
HEPATOTOXIC DRUGS 102
CONSEQUENCES OF LIVER FAILURE 104
PANCREATIC SECRETION (i) 104
PANCREATIC SECRETION (ii) 106
INSULIN AND GLUCOSE CONTROL 108
HEPATOBILIARY SYSTEM
AND PANCREAS
SECTION
3
1. The liver
a. Receives 25 per cent of its blood supply from the portal vein
b. Contains 50 000100 000 lobes
c. Has its own specialized macrophages
d. Is composed of functional units called acini
e. Receives 29 per cent of the cardiac output
2. True or false? The liver
a. Is innervated by the coeliac plexus
b. Develops from endodermal epithelium
c. Is more susceptible to ischaemic damage in the perivenous area of the acini
d. Can store up to 3 L of extra blood
e. Can perform haematopoiesis
3. The liver performs the following functions
a. Ca
2+
homeostasis
b. Formation of bile
c. Storage of vitamins
d. Gluconeogenesis
e. Insulin synthesis
Hepatobiliary system and pancreas 75
EXPLANATION: THE LIVER STRUCTURE AND FUNCTION
The liver is the largest gland in the body. It lies beneath the dome of the diaphragm and the central tendon
on the right side of the body.
It is composed of four lobes right, left, caudate and quadrate which contain within them 50000100000
lobules. A lobule can be thought of as a cylindrical structure of hepatic cells (hepatocytes) surrounding a
central vein. However the liver is sometimes described as being arranged into acini, with the portal triad at
the centre. This arrangement is used to emphasize the endocrine function of the liver.
The acini are divided into three zones to reflect the metabolic gradient of the liver. The centre of the acinus
(periportal area or zone 1) receives the most oxygenated blood. As blood filters peripherally, oxygen is used
up. Zone 3 is the least oxygenated. However, zone 1 is most susceptible to damage from ingested toxins via
the portal blood.
Innervation to the liver is by the hepatic plexus, a derivative of the coeliac plexus. The plexus contains
branches from the vagus nerves and the phrenic nerves. The nerves contain both sympathetic and parasym-
pathetic fibres.
The liver has numerous functions, principally to do with metabolism. Glucose and glycogen are both syn-
thesized in the liver, fatty acids are synthesized and packaged into triglycerides for transport in lipoproteins.
Cholesterol is also synthesized here. Important plasma proteins such as albumin and clotting factors are
formed, and amino acids are deaminated for the formation of urea. The liver also produces bile, acts as a store
for vitamin A, D and B12, and detoxifies drugs and foreign compounds.
Answers
1. F F T T T
2. T T T F T
3. F T T T F
Bile duct
Central vein
Branch hepatic
artery
Branch portal
vein
Portal
triad
Blood flow
Bile canaliculus
Zone 1 Zone 2 Zone 3
ONE STOP DOC 76
4. Regarding the anatomical lobes of the liver
a. They are approximately equal in size
b. The caudate lobe lies between the fissure for ligamentum venosum and the fossa for the
inferior vena cava
c. The falciform ligament divides the liver into functional left and right lobes
d. The bare area of the liver is in direct contact with the diaphragm
e. The gallbladder fossa lies in the quadrate lobe
5. Regarding the liver surface anatomy
a. The liver lies between ribs seven and eleven
b. The liver moves superiorly during inspiration
c. The inferior border follows the costal margin
d. The liver is located more inferiorly when erect
e. The liver is positioned such that it is not easily injured
6. Fill in the gaps in the following paragraph about the anatomy of the liver, using the
words from the list below. You may use each word once, more than once, or not at all
Options
A. Diaphragm B. Falciform ligament
C. Porta hepatis D. Visceral
E. Left F. Right upper quadrant
G. Bare area H. Parietal
I. Portal vein J. Hepatorenal recess
K. Quadrate L. Subphrenic recesses
The liver lies mainly in the 1, just below the inferior surface of the diaphragm. Much of the
anterior and superior parts of the liver are tucked under the 2, however, they are prevented from
contacting it by the 3. This diaphragmatic surface of the liver is entirely covered in 4 peritoneum
except for the 5, posteriorly; it begins where the peritoneum forms the coronary ligament. The
visceral surface of the liver is also covered with peritoneum except at the 6, a transverse fissure
between caudate and 7 lobes which allows entry of the hepatic artery and 8.
GB, gallbladder; IVC, inferior vena cava
Hepatobiliary system and pancreas 77
EXPLANATION: THE LIVER ANATOMY
The liver lies between ribs seven and eleven, in the right upper quadrant of the abdomen. The lower border
extends from the tip of the tenth rib to just below the left nipple. It may be palpable in a normal subject,
particularly on inspiration, when it moves inferiorly.
Recognizing boundaries for the lobes of the liver can be tricky since there are functional definitions and, of
course, anatomical definitions. The liver is divided into two equal sized functional lobes by an imaginary line
running through the gallbladder fossa and inferior vena cava. However, the right and left anatomical lobes of
the liver are demarcated based on the position of the lesser omentum (connecting liver to gut) and falciform
ligamen (attaches liver to anterior abdominal wall). Both these mesenteries are derived from the anterior
mesentery in the developing gut, which the liver grows into.
Answers
4. F T F T F
5. T F T T F
6. 1 F, 2 A, 3 L, 4 D, 5 G, 6 C, 7 K, 8 I
Right triangular
ligament
Ligamentum teres
Ligamentum
venosum
IVC Bare area
Portal vein
Portal triad is normally
enclosed in lesser omentum
Caudate lobe
Quadrate
lobe
GB
Visceral surface (posterior view)
Right triangular ligament
Coronary ligament
Left lobe Right lobe
Falciform ligament
Ligamentum teres
GB
Anterior view
ONE STOP DOC 78
7. True or false? The space of Disse
a. Lies between the endothelial cells of sinusoids and hepatic cells
b. Connects with lymphatic vessels
c. Transports bile to bile canaliculi
d. Contains large plasma proteins
e. Consists of large pores in the endothelial lining
8. Bile canaliculi
a. Run intracellularly in hepatic cords
b. Are sealed by tight junctions
c. Form part of a portal triad
d. Empty into canals of Hering
e. May transport blood
9. The hepatic portal vein
a. Has a pressure of about 10 mmHg
b. Drains blood from the liver to the inferior vena cava
c. Has no vasodilatory innervation
d. Supplies the peripheral portion of the acinus better than the centre
e. Blood does not mix with the hepatic artery blood
10. The portal vein
a. Is formed by the union of the splenic vein and left gastric vein
b. Contains blood which is poorly oxygenated
c. Contains blood which is nutrient-rich
d. Drains the entire gastrointestinal tract
e. Communicates with the systemic circulation via anastamoses
Hepatobiliary system and pancreas 79
EXPLANATION: LIVER MICROSTRUCTURE
Here we look in further detail at the microstructure of the liver. Networks of capillaries called sinusoids lie
between hepatocytes in the liver. The endothelial cells are separated from the hepatic cells by a narrow tissue
space called the space of Disse. Large pores in the endothelium allow plasma to move freely into the space.
In the fetus, the space of Disse contains haematopoietic cells.
Bile canaliculi are small canals which lie between adjacent hepatic cells. They receive bile from hepatocytes,
drain into canals of Hering, which then drain into bile ducts, which form part of the portal triad.
Branches of the hepatic portal vein form part of the portal triad together with the hepatic artery and bile
duct. It transports solutes from the gut to the liver for metabolism. Seventy-five per cent of the blood supply
to the liver is from the hepatic artery the blood mixes in the sinusoids. The hepatic portal vein also deliv-
ers blood (at a rate of 1 L/min) to the liver. It is formed from the splenic, superior mesenteric and inferior
mesenteric veins, in addition to other smaller tributaries just behind the neck of the pancreas. The portal vein
terminates at the porta hepatis (a transverse fissure between caudate and quadrate lobes), where it divides into
right and left branches. The portal venous system communicates with the systemic venous system via anasta-
moses at the oesophagus, the rectum, the anterior abdominal wall and the colon.
Answers
7. T T F T F
8. F T F T F
9. T F T F F
10. F T T T T
Fenestrated
endothelium
ITO cell
Hepatocyte
Hepatic cord
Branch of hepatic artery
Branch of portal vein
Endocrine secretion
Kupffer cell
(macrophage)
Sinusoid
Central vein
Reticular fibres in
the space of Disse
Bile canaliculus
groove between
hepatocytes
Tight junction
between cells
Bile duct
ONE STOP DOC 80
11. True or false? The following factors stimulate bile secretion by the liver
a. Increased hepatic blood flow
b. Contraction of the gallbladder
c. Vagal stimulation
d. Secretin
e. Low concentration of bile acid in blood
12. Bile has the following functions
a. To digest fat
b. To bind amino acids and aid their transport across the intestinal mucosal membrane
c. To neutralize acid chyme
d. To emulsify fat particles
e. To transport waste products away from the liver
13. Regarding bile acids
a. Their synthesis is catalysed by cholesterol 7 alpha hydroxylase
b. They are degraded in the terminal ileum
c. A deficiency causes malabsorption of fat
d. They are bound by resins to reduce hyperlipidaemia therapeutically
e. They may be conjugated to form bilirubin
14. True or false? The bile duct
a. Is formed by the union of the pancreatic duct and the cystic duct
b. Varies from 5 to 15 cm in length
c. Descends posterior to the first part of the duodenum
d. Releases bile directly into the large intestine
e. Opens in the wall of the gut at the ampulla of Vater
Hepatobiliary system and pancreas 81
EXPLANATION: BILE SECRETION
Bile contains water, bile acids, cholesterol, lipids and the bile pigments bilirubin and biliverdin.
Around 500 mL of bile is secreted by the liver per day. It helps to emulsify large fat particles so that they can
easily be attacked by the lipase enzyme secreted in the pancreatic juice. It also aids their transport through the
intestinal mucosal membrane and removes waste products (including bilirubin) from the liver.
The main bile pigment, bilirubin, comes from the breakdown of haemoglobin in old red blood cells. This
takes place in the reticuloendothelial system. Bilirubin is then transported to the liver by the circulation bound
to albumin, since it is insoluble.
Bile acids are synthesized in the hepatocytes themselves. They are all formed from cholesterol. Bile salts are
then formed from these acids by conjugation with K
+
or Na
+
. Primary bile salts are formed in the liver. They
are then modified by bacteria in the colon to form secondary bile salts.
The hepatocytes secrete these components in the form of bile into the small intestine where 9095 per cent
of the bile salts/acids are reabsorbed (mostly in the terminal ileum) and enter the portal vein to be transported
back to the liver for recycling. This is known as enterohepatic recirculation. For this reason we only synthe-
size a small amount of bile acids.
The bile pigments bilirubin and biliverdin are normally excreted in the faeces.
Factors that increase the secretion of bile by the liver are known as choleterics. They include stimulation by
the vagus nerve, secretin and, most importantly, the bile salts themselves.
Answers
11. T F T T F
12. F F F T T
13. T F T T F
14. F T T F F
ONE STOP DOC 82
15. The gallbladder
a. Contracts in response to cholecystokinin
b. Holds a maximum of 500 mL of bile
c. Empties by contraction of the sphincter of Oddi
d. Is stimulated by the vagus nerve
e. Concentrates bile up to 15 times
16. With regard to the gallbladder
a. It lies in the cystic fossa of the liver
b. It has a capacity of up to 50 mL of bile
c. It is supplied by a branch of the splenic artery
d. It contacts the ascending colon
e. It receives a parasympathetic nerve supply from the vagus
17. Label the diagram below as indicated
Options
A. Pancreatic duct
B. Left hepatic duct
C. Fundus
D. Neck
E. Liver
F. Pancreas
G. Hepato-pancreatic ampulla
H. Bile duct
I. Common hepatic duct
J. Cystic duct
18. Concerning the gall bladder
a. What causes the gallbladder to contract?
b. How is the composition of bile changed in the gallbladder?
c. Name three factors that help the sphincter of Oddi to relax
CCK, cholecystokinin; DD, duodenum; GB, gallbladder
6
1
2
3
4
5
9
10
8
DD
GB
7
Hepatobiliary system and pancreas 83
EXPLANATION: THE GALLBLADDER
The gallbladder has three parts, the fundus, body and neck. The fundus lies at the margin of the liver, and
the body lies against the transverse colon, in front of the first part of the duodenum. It narrows into the neck
from which the cystic duct opens. The gallbladder is supplied by the cystic artery, which is usually a branch
of the right hepatic artery (from the common hepatic artery). However, variations can occur in its origin and
course.
The bile duct is formed by the union of the cystic duct (from the gallbladder) and the common hepatic duct
(from the liver). It transports bile to the wall of the second part of the duodenum (the descending bit). Here,
it is joined by the pancreatic duct at the ampulla of Vater, and opens into the duodenum at the duodenal
papilla.
The maximum volume of the gallbladder is 3060 mL. However, bile is concentrated within the bladder by
the absorption of water, Na
+
and Cl

into the gallbladder mucosa (18b). Up to 12 hours of bile secretion is


stored in concentrated form. Factors that cause contraction of the gallbladder are known as cholagogues
(18a). They include the entrance of food into the mouth, impulses from the vagus nerve and the secretion
of CCK. Remember CCK is secreted in response to fatty acids and amino acids in the duodenum, as well as
acid and Ca
2+
.
The gallbladder starts emptying about 30 minutes after a meal is eaten. The walls contract rhythmically, and
the sphincter of Oddi which guards the entrance to the duodenum relaxes. Relaxation of the sphincter of
Oddi is assisted by: (1) peristaltic waves transmitted down the bile duct from the gallbladder, (2) cholecys-
tokinin, and (3) intestinal peristaltic waves in the duodenum wall (18c).
Answers
15. T F F T T
16. T T F F T
17. 1 D, 2 J, 3 E, 4 B, 5 I, 6 H, 7 C, 8 G, 9 A, 10 F
18. See explanation
ONE STOP DOC 84
19. Case study
An overweight 46-year-old smoker was admitted to the Emergency Department with a sudden
onset of severe pain in the right upper quadrant of her abdomen, vomiting and fever. On
palpation of her abdomen there was tenderness and guarding in the right upper quadrant,
which became worse on inspiration. Ultrasound revealed a thickened gallbladder wall and
pericholecystic fluid.
a. What are the risk factors for gallstones?
b. Why does this woman complain of pain in the shoulder?
c. What complications of this condition could arise?
Hepatobiliary system and pancreas 85
EXPLANATION: GALLSTONES
Gallstones are most likely to affect you if you are:
Fair
Forty
Fat
Fertile
Female
In other words, an obese middle-aged woman who has had several children (remember the five Fs) (19a).
Smoking also increases the risk that gallstones will be symptomatic, although 50 per cent are not. Gallstones
are generally more common in Europe and North America than in the rest of the world.
There are three types of stone: Ca
2+
bilirubinate, cholesterol and mixed. Predisposing factors to their forma-
tion are: bile stasis, supersaturation of bile with cholesterol (due to excessive cholesterol excretion), and a mix
of nucleation factors that favours stone formation. If gallstones become impacted in the gallbladder, cystic duct
or bile duct they can cause a characteristic pain known as biliary colic. Pain may be referred to the right
shoulder because the inflamed gallbladder may irritate the peritoneum covering the diaphragm (19b). The
skin of the shoulder is supplied by the same segments of the spinal cord that receive pain afferents from the
diaphragm (C3, C4).
Complications of gallstones are numerous and depend on the location of the stone. Within the gallbladder,
acute cholecystitis (inflammation and oedema of the gallbladder) may arise, empyema (pus in the gallblad-
der) and even perforation may occur (19c). Organisms associated with cholecystitis are Escherichia coli,
Klebsiella and Streptococcus faecalis. Repeated bouts of acute cholecystitis may result in chronic cholecystitis,
with fibrosis of the gallbladder wall.
In the bile ducts, obstructive jaundice, pancreatitis or cholangitis (inflammation of the bile duct) may occur.
And in the gut, there is a risk of gallstone ileus a form of intestinal obstruction.
In this case it is likely the woman has acute cholecystitis because of the sudden onset, the fever and local peri-
tonitis. The pain felt on inspiration is known as Murphys sign. She would be treated initially with pain relief,
fluids and antibiotics, then cholecystectomy.
Answers
19. See explanation
ONE STOP DOC 86
20. True or false? The insulin receptor
a. Contains two alpha subunits and two beta glycoprotein subunits
b. Has a tyrosine kinase domain on the intracellular ends of the alpha chain
c. Can be downregulated by a high concentration of insulin
d. Number is increased in the fed state
e. Does not always require autophosphorylation to exert its biological effects
21. Regarding the insulin receptor
a. It is a hexamer
b. The beta subunit binds insulin
c. Insulin receptor substrate mediates the actions of insulin
d. The beta subunit autophosphorylates on serine/threonine residues
e. Receptors may only be found on adipocytes
22. Label the indicated parts of the insulin receptor opposite
23. State whether the following metabolic processes are (1) increased, (2) decreased or
(3) unaffected by insulin
Options
A. Lipolysis
B. Glucose transport into muscle cells
C. Glycogen breakdown
D. Conversion of glucose to fatty acids
E. Glucose uptake into brain cells
F. Active transport of amino acids into cells
Hepatobiliary system and pancreas 87
EXPLANATION: THE INSULIN RECEPTOR AND THE EFFECTS OF INSULIN
The insulin receptor is a single polypeptide arranged into a tetramer of alpha and beta subunits linked by
disulphide bonds (it belongs to a group of receptors called tyrosine kinase receptors). The receptors are found
on all tissues including the liver. Insulin binds to the extracellular alpha subunits, which triggers ATP gener-
ation to cause autophosphorylation of the tyrosine residues on the membrane-spanning beta subunits. The
beta subunits have a tyrosine kinase domain, and therefore following autophosphorylation they become acti-
vated tyrosine kinases which phosphorylate tyrosine residues on IRS, an important intracellular mediator of
insulins actions. The phosphorylation of IRS initiates its own catalytic activity and results in a phosphoryla-
tion/dephosphorylation cascade of intracellular proteins on serine and threonine residues which control the
biological actions of insulin.
Insulin is essentially an anabolic hormone. It has stimulatory and inhibitory effects in the liver, muscle and
adipose tissue all insulin-sensitive tissues. In hepatocytes, insulin causes an increase in glycolysis, glycogen
synthesis, fatty acid and triacylglycerol synthesis, and inhibits gluconeogenesis. In adipocytes, glucose uptake
by GLUT-4 transporters is increased, as is lipogenesis. Insulin inactivates hormone-sensitive lipase so that
breakdown of fat is inhibited. In muscle, again glucose uptake by GLUT-4 is increased, as are glycolysis, glyco-
gen synthesis and protein synthesis. Glycogen degradation and proteolysis are inhibited. The brain and ery-
throcytes do not modify their glucose uptake in response to insulin.
Answers
20. T F T F F
21. F F T F F
22. 1 alpha subunit; 2 cysteine residues; 3 disulphide bridge
23. 1 B, D, F; 2 A, C; 3 E
NH
2
Extracellular
binding
domain
Membrane
COOH COOH
Cytoplasmic tyrosine
kinase domain
Transmembrane domain
NH
2
NH
2

1 2
3
3
3
1
ONE STOP DOC 88
24. Regarding development of the liver
a. Growth begins in the middle of the third week of development
b. The liver is derived from ectodermal epithelium
c. The gallbladder and liver originate from the same liver bud
d. Haematopoiesis begins during the third week of development
e. The liver grows slowly during embryonic life
25. Regarding hepatobiliary development
a. The liver is derived from the midgut
b. The septum transversum is a mass of endodermal cells
c. The cranial part of the liver bud gives rise to the gallbladder
d. Bile formation begins in the 12th week
e. Biliary atresia results from the failure of the bile ducts to recanalize
26. Regarding the development of the pancreas
a. The pancreas develops in the 4th and 6th week of fetal life
b. The ventral bud migrates in front of the duodenum
c. The ventral bud and dorsal bud do not fuse until after birth
d. The accessory pancreatic duct is derived from the dorsal pancreatic bud
e. Insulin secretion begins at birth
Hepatobiliary system and pancreas 89
EXPLANATION: DEVELOPMENT OF THE LIVER AND PANCREAS
The liver, the gallbladder and the bile duct all arise in the middle of the 3rd week to early in the 4th week of
development from a ventral outgrowth of endodermal epithelium in the foregut called the liver bud or
hepatic diverticulum.
The liver bud extends into the septum transversum, a mass of mesodermal cells between the pericardial cavity
and the yolk sac. Septum transversum eventually forms the diaphragm and some ventral mesentery.
The liver bud actually divides, so that the cranial part forms the cells of the liver parenchyma and the caudal
(inferior) part forms the gallbladder and cystic duct.
The liver grows very rapidly, intermingling with the umbilical and vitelline veins. Haematopoiesis begins
during the 6th week, and bile formation by the hepatocytes begins in the 12th week.
The pancreas develops from two outgrowths of the endodermal epithelium, again in the foregut. They are the
ventral pancreatic bud and the dorsal pancreatic bud.
The ventral pancreatic bud rotates and migrates round the back of the duodenum to join the dorsal bud, where
the two buds fuse. Their respective ducts also fuse to form the main pancreatic duct that opens into the duo-
denum with the common bile duct. The accessory pancreatic duct which may be found in some individuals
is actually a persistent part of the dorsal bud duct.
The ventral bud forms the head of the pancreas, whilst the dorsal bud forms the body and tail. Insulin is
secreted during the 5th month of embryonic life.
Answers
24. T F T F F
25. F F F T T
26. T F F T F
At six weeks
Liver
Stomach
Tail of pancreas
Head of pancreas
(ventral bud has swung round
to fuse with dorsal bud)
Duodenum
Developing gallbladder
ONE STOP DOC 90
27. True or false? Insulin
a. Is secreted into the bloodstream via the main pancreatic duct
b. Has intramolecular disulphide bridges
c. Is stored with C-peptide in granules
d. Cannot be broken down
e. Is formed from an inactive precursor
28. Insulin secretion is stimulated by
a. Adrenaline b. A rise in blood glucose
c. Secretin d. Glycerol
e. A rise in amino acid in the blood
29. In an oral glucose tolerance test, impaired glucose tolerance
a. Is demonstrated by a greater increase in plasma glucose than normal
b. Is demonstrated by a faster drop in plasma glucose to baseline than normal
c. Is caused by increased intestinal glucose absorption
d. Impairs the ability of the brain and red blood cells to take up glucose
e. Is diagnostic of diabetes mellitus
30. Regarding the oral glucose tolerance curve
a. On the adjacent graph, draw the
glucose tolerance curve for: (i) a
normoglycaemic person and (ii) a
person with insulin-dependent
diabetes
b. What is the normal range for fasting
blood glucose?
75g Glucose
Glucose tolerance curve
Hours
1 0 2 3 4 5
80
100
120
140
10
5
180
160
200
B
l
o
o
d

g
l
u
c
o
s
e

(
m
g
/
d
l
)
B
l
o
o
d

g
l
u
c
o
s
e

(
m
m
o
l
)
Hepatobiliary system and pancreas 91
EXPLANATION: INSULIN
Insulin is a polypeptide hormone produced by beta-cells of the islets of Langerhans. Beta-cells make up
around 1 per cent of the mass of the pancreas. Insulin has anabolic effects on the peripheral tissues, pro-
moting uptake of glucose, protein synthesis, lipogenesis, glycogen synthesis and growth.
Fifty-one amino acids are arranged into two polypeptide chains A and B linked by disulphide bridges.
There are also intramolecular disulphide bridges between amino acid residues 6 and 11 of the A chain.
Insulin is synthesized from an inactive precursor. Preproinsulin has an N-terminal signal peptide cleaved in
the rough endoplasmic reticulum to give proinsulin, which is transported to the Golgi where it is cleaved to
give insulin and C-peptide which are stored in secretory granules in the cytosol. Secretion is by exocytosis on
initiation by the appropriate stimulus.
Stimulate insulin secretion Inhibit insulin secretion
Rise in blood glucose Low blood glucose
Rise in plasma amino acids, especially Adrenaline (released in response to
arginine stress/trauma/extreme exercise)
Secretin (GI hormone released in
response to ingestion of food)
Glucagon
The oral glucose tolerance test involves taking 75 g
of glucose in 300 mL water after fasting overnight.
Venous plasma glucose is measured before the drink
and over the next 2 hours. An individual with
impaired glucose tolerance will have a slightly
higher than normal fasting plasma glucose, and their
plasma glucose will be between 6.0 and 7.8 mmol/L
2 hours after intake (30). If fasting plasma glucose is
>7.8 mmol and plasma glucose remains >11.1
mmol 2 hours after intake, the individual is
diabetic.
Answers
27. F T T F T
28. F T T F T
29. T F F F F
30. See explanation
75g Glucose
Glucose tolerance curve
Hours
1 0 2 3 4 5
80
100
120
140
10
5
180
160
200
B
l
o
o
d

g
l
u
c
o
s
e

(
m
g
/
d
l
)
B
l
o
o
d

g
l
u
c
o
s
e

(
m
m
o
l
)
Diabetic
Normal
ONE STOP DOC 92
31. Match the following features of acute liver failure to the causes listed below
Options
A. Yellow skin
B. Prone to bleeding
C. Lethargy and drowsiness
D. Low glomerular filtration rate
E. Raised aspartate aminotransferase and alanine aminotransferase
1. Failure of liver to synthesize proteins, depletion of factors II, VII, IX and X
2. Circulation of excitatory amino acids, failure of liver to detoxify nitrogenous compounds
3. Dysfunctional bilirubin metabolism
4. Hepatocyte necrosis releases enzymes into bloodstream
5. Shock due to a low circulating blood volume
32. Fatty changes to the liver
a. Are caused by the accumulation of fat outside the liver
b. May be caused by starvation
c. May occur in pregnancy
d. Are always irreversible
e. Are usually lethal
GFR, glomerular filtration rate
Hepatobiliary system and pancreas 93
EXPLANATION: LIVER FAILURE
Acute liver failure occurs when there has been damage to the majority of the hepatocytes in the liver, such
that its functions are impaired. It may result from metabolic damage, severe systemic shock or as an acute
decline in chronic liver disease. Acute liver failure has several key features.
Clinical jaundice is the yellowing of skin and sclerae caused by a high plasma bilirubin (above 50 mol/L).
This occurs in acute liver failure due to dysfunctional bilirubin metabolism. The liver is responsible for the
conjugation and secretion of bilirubin in bile to the gut. Failure to do either results in accumulation of biliru-
bin in the blood.
Hepatic encephalopathy may be caused by acute liver failure. Features are progressive drowsiness, lethargy
and eventually coma. It is caused by failure of the liver to detoxify nitrogenous compounds and the circula-
tion of excitatory amino acids (which are not being deaminated in the liver).
Enzyme levels in the blood, such as aspartate aminotransferase and alanine aminotransferase, are useful
clinical indicators of hepatocellular damage as they leak into the circulation.
Patients with liver failure may develop kidney failure (hepatorenal syndrome). Despite the almost normal his-
tological appearance of the kidney, GFR is low, is as Na
+
concentration of the urine.
Fatty change is just one pattern of histological abnormality seen in the liver following damage. Triglycerides
accumulate within the liver cells. It is most commonly seen when the liver is exposed to metabolic stress and
alcohol. It is sublethal and, in the case of alcohol-induced fatty change, is reversible by abstinence.
The main causes of fatty liver are hypoxia, starvation, diabetes mellitus, alcohol, Reyes syndrome and preg-
nancy.
Other patterns of pathological response seen on damage to the liver are:
Liver cell necrosis
Fibrosis
Cholestasis
Storage of abnormal material.
Answers
31. 1 B, 2 C, 3 A, 4 E, 5 D
32. F T T F F
ONE STOP DOC 94
33. Concerning hepatitis
a. Hepatitis A may lead to chronic liver disease
b. Hepatitis B is transmitted by the faecaloral route
c. It is possible to carry hepatitis B and not show symptoms
d. It may be caused by the EpsteinBarr virus
e. Hepatitis C decreases the risk of hepatocellular carcinoma
34. Regarding acute hepatitis
a. Symptoms are low-grade pyrexia and malaise
b. It is not associated with jaundice
c. The cause is usually bacterial
d. Liver cells swell and become vacuolated
e. Recovery usually takes between 3 and 8 weeks
AST, aspartate aminotransferase; ALT, alanine aminotransferase
Hepatobiliary system and pancreas 95
EXPLANATION: HEPATIC PATHOLOGY
There are six main patterns of liver cell necrosis:
1. Piecemeal necrosis 2. Massive necrosis 3. Zonal necrosis
4. Bridging necrosis 5. Councilman bodies 6. Spotty necrosis.
Spotty necrosis is usually seen with toxic damage and viral infection. In zonal necrosis, certain zones are
affected by certain diseases, for example zone 3 is affected by paracetamol, whereas zone 1 is affected by phos-
phorous toxicity. Following liver cell damage there is regeneration of the liver cells to restore hepatic func-
tion. In cases of chronic damage, regeneration may be disturbed by fibrosis. ITO cells differentiate to secrete
collagen in the space of Disse. This is the cause of liver cirrhosis.
Hepatitis is the term used to refer to inflammatory diseases of the liver. It may be acute or chronic. The main
causes are viral, autoimmune diseases, drug reactions and alcohol. Viruses are a particularly common cause
of hepatitis. Clinical features of acute hepatitis are nausea, anorexia, low-grade pyrexia and general malaise.
Jaundice occurs a week to 10 days after the onset of symptoms. Serum bilirubin is usually raised, and levels
of ALT and AST are very high early on in the disease. They fall as the liver regenerates itself. Recovery is within
38 weeks. The main hepatitis viruses are A and E transmitted via the faecaloral route and B, C and D,
which are transmitted via the parenteral route. EpsteinBarr virus and cytomegalovirus also cause hepatitis.
The table below shows some of the important features of each virus:
Answers
33. F F T T F
34. T F F T T
Hepatitis Virus type Transmission route Clinical features Chronicity
A RNA Faecaloral Fever, malaise and nausea + vomiting No
within 2 weeks
Jaundice 1 week later
Nearly all patients fully recover
Incubation 15 months
B DNA Parenteral, sexual, Asymptomatic or similar to hepatitis A Yes
vertical (mother to
baby)
95% fully recover 5% chronic carriers
1% fulminant liver failure risk of hepatocellular
carcinoma
C RNA Parenteral Acute infection is mild Yes
20% jaundice 50% chronic hepatitis
50% recover fully risk of hepatocellular
carcinoma
D RNA. Parenteral seen in Illness is indistinguishable from No
Requires presence i.v. drug users, in hepatitis B
of HBV to infect particular
Incubation 1 month
E RNA Faecaloral Mild infection associated with jaundice No
Mortality from fulminant liver failure is 1%
ONE STOP DOC 96
35. True or false? The following are histological features of liver cirrhosis
a. Councilman bodies
b. Fibrosis
c. Destruction of liver cells
d. Lymphoid cells in the portal tracts
e. Regeneration of hepatocytes to form nodules
36. Portal hypertension is caused by
a. Splenomegaly
b. Certain herbal teas
c. Liver cirrhosis
d. Portal vein thrombosis
e. Haemorrhoids
Hepatobiliary system and pancreas 97
EXPLANATION: CIRRHOSIS OF THE LIVER
In liver cirrhosis the normal architecture of the liver is replaced by diffuse nodules of regenerated liver cells,
separated by bands of collagenous fibrosis. Cirrhosis is irreversible. Two types are described histologically:
micronodular, where the nodules are less than 3 mm in diameter, and macronodular, where the nodules are
larger than 3 mm in diameter.
Alcohol is the most common cause of cirrhosis in the western world. It is associated with micronodules.
Another common cause is chronic hepatitis (B and C), which is associated with macronodules. Less common
causes are cystic fibrosis, biliary cirrhosis and autoimmune disease. The clinical features of liver cirrhosis are
secondary to portal hypertension and liver cell failure.
The portal vein accounts for 85 per cent of hepatic blood flow. Obstruction to flow from the portal vein
causes a build up of pressure in the portal vascular bed, resulting in splenomegaly, ascites and portosystemic
anastamosis. Causes of portal hypertension can be pre-sinusoidal, sinusoidal or post-sinusoidal.
Pre-sinusoidal occlusion of the portal venous system before the portal tracts, for example portal vein
thrombosis, predisposed to by local sepsis and polycythaemia.
Sinusoidal blockage in the sinusoids caused by hepatic diseases such as cirrhosis, portal tract fibrosis or
polycystic disease of the liver.
Post-sinusoidal diseases that block central veins and venous branches. For example thrombosis of hepatic
or central veins, scarring caused by alcohol abuse, cytotoxic drugs, ingestion of toxic alkaloids present in
herbal teas, venous fibrosis caused by irradiation of the liver.
BuddChiari syndrome is caused by occlusion of the hepatic vein. Patients develop massive vascular con-
gestion of the liver, which enlarges grossly. They also develop ascites very rapidly and jaundice. It can result in
rapid death.
Answers
35. F T T F T
36. F T T T F
Thrombosis of
hepatic vein/central vein
Sinusoidal
Pre-sinusoidal
Post-sinusoidal
Portal vein
thrombosis
Hepatic artery
Hepatic
portal vein
Bile duct
Liver
Hepatic disease:
Cirrhosis
Portal tract fibrosis
Polycystic disease
ONE STOP DOC 98
37. Regarding the pancreas
a. The head rests on the inferior vena cava
b. Pancreatic juice is secreted into the bile duct
c. The body lies at the level of L5
d. The pancreatic duct and common bile duct unite to form the ampulla
e. The tail lies between the layers of the splenorenal ligament
38. Pancreatic cancer
a. Usually affects the head of the pancreas
b. Causes obstructive jaundice
c. Is not associated with smoking
d. Presents with pain in the chest
e. Does not cause diabetes
39. Case study
A 45-year-old man is brought into the Emergency Department in agony, experiencing severe
epigastric pain radiating through to the back, feeling nauseous and retching violently. The pain
started 2 hours previously as he was returning from a meal out at a restaurant. On examination
the man appeared jaundiced, was feverish and tachycardic.
Blood results showed a high white cell count, hyperglycaemia, a low plasma Ca
2+
and a high
level of amylase in the plasma.
a. What is the likely cause of this mans abdominal pain?
b. Why is he hyperglycaemic?
c. Why does he have a low plasma Ca
2+
?
Hepatobiliary system and pancreas 99
EXPLANATION: THE PANCREAS
The pancreas produces an exocrine secretion that enters the duodenum through the pancreatic duct and an
endocrine secretion that enters the blood directly. The bile duct joins the pancreatic duct and carries only bile.
The head of the pancreas is hugged by the C-curve of the duodenum. It rests on the inferior vena cava, the
right renal artery and vein, and the left renal vein. The body lies at the level of L2.
Pancreatic cancer is the fifth most common cause of death in the western world. Cancers are invariably ductal
adenocarcinomas. Smoking causes a two-fold increased risk, and 60 per cent of patients are male. Other risk
factors are thought to be high-fat, high-protein diets, alcohol and coffee. Obstructive jaundice is caused by
the compression of the bile duct by the tumour, together with dark urine and pale stools. Pancreatic cancer
often presents late as an ill-defined pain radiating to the back, partially relieved by sitting up. Exocrine and
endocrine insufficiency are also associated diabetes mellitus or impaired glucose tolerance is present in one-
third of patients. Weight loss (reflecting malabsorption, low dietary intake and depressed liver function),
steatorrhoea and anorexia are also features. Surgical resection is the only hope for cure. The standard opera-
tion is Whipples procedure removal of the pancreas, distal stomach, duodenum, gallbladder and common
bile duct, and then anastamoses between the jejunum and pancreatic remnant.
The man in the case study has acute pancreatitis, a very common reason for emergency admission to hospi-
tal (39a). Major causes are biliary tract disease and heavy alcohol consumption. It may be precipitated by
a fat-rich meal, causing hyperlipidaemia. The severity may range from mild oedema to severe necrosis and
haemorrhage. The exact pathophysiology of pancreatitis remains unclear. It seems that a triggering factor
causes premature activation of pancreatic enzymes within the pancreatic ducts, setting off a chain reaction of
cell necrosis and autodigestion. Proteolytic enzyme release results in increased capillary permeability, protein
exudation and oedema. The fluid loss causes hypovolaemic shock, which may be dangerous. Destruction of
the islets of Langerhans leads to a transient insulin deficiency and therefore hyperglycaemia prevails (39b).
Liver function enzymes may be raised in the blood in alcoholic patients. Plasma amylase, a pancreatic diges-
tive enzyme, is very high due to its release from damaged pancreatic cells. Another enzyme released is pancre-
atic lipase. It breaks down retroperitoneal fat to free fatty acids, which then absorb Ca
2+
to form insoluble Ca
2+
salts. This is what causes the low plasma Ca
2+
(39c).
Answers
37. T F F T T
38. T T F F F
39. See explanation
ONE STOP DOC 100
40. True or false? Bilirubin
a. Is formed from the degradation of haemoglobin
b. Is transported free in the plasma
c. Is conjugated in the kidney
d. May undergo enterohepatic recirculation
e. Is excreted in the urine as stercobilinogen
41. Jaundice
a. Is characterized by a high concentration of bilirubin in the blood
b. Causes itching of the skin
c. Is a sign of liver disease
d. May be caused by gallstones impacted in the bile duct
e. Is cured by removal of the gallbladder
42. Define the following terms
a. First pass metabolism
b. Phase I reaction
c. Phase II reaction
sER, smooth endoplasmic reticulum; GB, gallbladder; RBC, red blood cell
Hepatobiliary system and pancreas 101
EXPLANATION: JAUNDICE AND DRUG METABOLISM
The bilirubin pathway is shown on page 102.
Jaundice is caused by abnormalities either in bilirubin metabolism in the liver or in bilirubin excretion,
resulting in an elevation in its level in the blood (3060 mol/L). It is characterized by yellowing of the skin
and cornea, and pruritus (itching). The liver normally conjugates bilirubin to make it more water soluble and
to ensure it is excreted in the urine. If the liver is unable to do this, bilirubin builds up in the plasma, causing
prehepatic jaundice:
Haemolysis too much bilirubin is formed
Gilberts syndrome abnormalities in bilirubin handling by liver
CriglerNajjar syndrome liver cells have defect in the enzyme that conjugates bilirubin.
Cholestatic/obstructive jaundice is caused by a blockage in the excretion of conjugated bilirubin from the
liver to the small intestine. Conjugated bilirubin builds up in the blood. Since it is water soluble, it is excreted
in the urine and turns it dark. However, the stools are pale since bile is absent from the faeces. Cholestasis may
be either intrahepatic (hepatitis, cirrhosis, pregancy-associated, hereditary enzyme defects) or extrahepatic
(gallstone in the bile duct, carcinoma of head of pancreas).
Some drugs are inactivated in the liver. They are removed from the portal circulation and metabolized.
Therefore the concentration of the drug reaching the systemic circulation is much less than that in the portal
vein. This is known as first pass metabolism (42a). Examples of drugs that undergo first pass metabolism
include aspirin, lignocaine, morphine and propanolol.
The disadvantages of first pass metabolism are that a much larger dose of the drug is required orally than via
other routes. Also, there is a marked individual variation in the extent to which drugs are metabolized like this,
so there is some unpredictability in determining how much drug will reach the systemic circulation.
Metabolism of drugs in the liver involves two kinds of reaction: phase I and phase II. Phase I is usually oxi-
dation, reduction or hydrolysis (42b). The products of the reaction usually have a reduced biological activ-
ity compared with the original drug, but may be more toxic. The aim of this reaction is to give the drug a reac-
tive chemical handle, so that it may proceed to phase II.
Phase II reaction involves the conjugation of a drug molecule with a suitably reactive handle (42c).
Conjugation is basically the attachment of a substituent group. The new conjugate is pharmacologically inac-
tive and more water soluble than its precursor, therefore it is readily excreted in the urine.
Answers
40. T F F T F
41. T T T T F
42. See explanation
ONE STOP DOC 102
43. The following drugs undergo first pass metabolism. True or false?
a. Sulphonureas b. Aspirin
c. Oral contraceptive pill d. Propanalol
e. Metronidazole
44. The following drugs are hepatotoxic
a. Halothane b. Androgens
c. Paracetamol d. Alcohol
e. Isoniazid
45. Theme chemical pathology of the liver. Match the following laboratory test results to
their respective indications
Options
A. Liver cell necrosis B. Cholestasis
C. Alcohol-induced damage D. Chronic liver disease
E. Acute hepatitis
1. Raised serum albumin 2. Extremely high aspartate aminotransferase
3. Raised alkaline phosphatase 4. Prolonged prothrombin time
5. Raised gamma glutamyl transpeptidase
GGT, gamma glutamyl transferase; AST, aspartate aminotransferase
Reticuloendothelial
system
Haemoglobin
Globin
RBCs
Haem
Fe Biliverdin
Bilirubin
Bilirubin
Bilirubin
bound to
albumin in
bloodstream
Bilirubin diglucuronide
(conjugated)
Liver
sER
Kidney
Excreted
in urine
Reabsorbed
into blood
Gut
GB
Conjugated
bilrubin
Unconjugated
bilirubin
Urobilinogen
Stercobilinogen
Faeces
Reabsorbed into portal blood
and recycled in the liver (enterohepatic circulation)
The bilirubin pathway (see Question 40, page 100).
Hepatobiliary system and pancreas 103
EXPLANATION: HEPATOTOXIC DRUGS
Many drugs may cause liver cell damage, which presents as hepatitis or cholestasis.
Paracetamol and isoniazid cause hepatocyte necrosis due to their highly reactive metabolites, which may
form covalent or non-covalent bonds with cell components. Covalent bonds are formed with DNA, proteins,
lipids and carbohydrates. Non-covalent interactions include lipid peroxidation, generation of toxic free radi-
cals and reactions depleting glutathione (an important participant in phase II reactions). Chlorpromazine and
androgen steroids cause a different type of liver cell damage which results in obstructive jaundice. Some drugs
cause an autoimmune reaction in the liver. Halothane is thought to damage liver cells by this mechanism.
Paracetamol toxicity is clinically very important, as it is a common cause of death from self-poisoning.
Paracetamol is usually very safe when taken in the correct doses. Normally it is metabolized in the liver by two
routes of conjugation. However, in massive doses the enzymes catalysing the conjugation reactions are satu-
rated, and the toxic reactive intermediate in the pathway accumulates (N-acetyl-p-benzoquinoneimine). The
intermediate causes liver cell death. Fulminant liver failure usually develops within 3648 hours due to
massive necrosis.
The liver function tests and their meanings are listed below.
Answers
43. F T T T F
44. T T T T T
45. 1 D, 2 A, 3 B, 4 E, 5 C
Test Normal range Results Interpretation
Serum bilirubin <17 mol/L unconjugated Pre-hepatic jaundice
conjugated Intrahepatic or extrahepatic cholectasis
Alkaline phosphatase enzyme 90300 iu/L Raised Bone disease or liver disease:
found in bone, gut, placenta particularly indicative or cholestasis
and liver, especially bile because enzyme is released proximal to
canalicular region the obstruction and refluxes into blood
GGT Men: up to 51 iu/L Raised GGT synthase is stimulated by alcohol
Women: up to 33 iu/L and other drugs
Albumin 3055 g/L Lowered Albumin has long half-life (21 days)
therefore is a good indicator of chronic
liver disease affecting synthesis
AST found in liver and muscle <50 iu/L Very high Severe hepatis: necrosis
Moderately high Membrane damage, no necrosis
Prothrombin time Control 4 s Longer than Prothrombin time depends on synthesis
normal of clotting factors. These have short half-
lives. Good indicator of acute hepatitis
because affected quickly
ONE STOP DOC 104
46. Explain the mechanism by which the following occur in liver failure
a. Ascites
b. Gynaecomastia
c. Hypoglycaemia
47. The following are signs of chronic liver disease. True or false?
a. Exophthalmia
b. Flapping tremor
c. Ascites
d. Aphthous ulcers
e. Spider naevi
48. Theme the pancreas. Match the following exocrine pancreatic secretions to their
correct action
Options
A. Pancreatic lipase B. Trypsin
C. Bicarbonate D. Pancreatic alpha-amylase
E. Phospholipase A
2
F. Elastase
1. Removes fatty acid from carbon 2 of phospholipid
2. Removes fatty acids from carbon 1 and 3 in triacylglycerides
3. Digests proteins and polypeptides
4. Neutralizes gastric acid to pH 6 or 7
5. Breaks down elastin
6. Breaks down starch
49. The following factors increase pancreatic exocrine secretion
a. Sympathetic stimulation b. Secretin
c. Gastrin d. Somatostatin
e. Vagal stimulation
Hepatobiliary system and pancreas 105
EXPLANATION: CONSEQUENCES OF LIVER FAILURE (QUESTIONS 46 AND 47)
The consequences of liver failure are all related to the deterioration of the main functions of the liver.
Ascites is related to a reduction in the synthesis of plasma proteins such as albumin and clotting factors (46a).
Hypoalbuminaemia causes a loss of oncotic pressure across the capillary walls. This is the osmotic force
exerted by plasma proteins to pull water into the blood. With a loss of this force, fluid escapes from the cir-
culation into the interstitial space, resulting in peripheral oedema. Fluid accumulating in the peritoneal cavity
is known as ascites.
With liver failure, hormones tend to accumulate in the bloodstream.
Aldosterone Na
+
and water retention
Oestrogen gynaecomastia in men and loss of secondary sex characteristics (46b)
Insulin reduced glycogen breakdown, impaired gluconeogenesis, and therefore lowered blood glucose
(46c).
Cirrhosis or chronic liver disease resulting in irreversible changes to the liver has a variety of physical signs that
can be seen in the patient on close examination. Spider naevi appear on the surface of the skin as a central
arteriole from which a series of smaller vessels branch. They are formed by local vascular dilation due to high
oestrogen levels and commonly occur on the neck, face and dorsa of the hands. Other physical signs are club-
bing of the finger nails, a flapping tremor caused by hepatic encephalopathy, loss of body hair, bruising due
to deficiency in clotting factors, a swollen abdomen due to the fluid retention of ascites, and xanthelasmata
cholesterol deposition around the eyelids. Aphthous ulcers are a feature of inflammatory bowel disease;
exophthalmia is seen in hyperthyroidism.
EXPLANATION: PANCREATIC SECRETION (i)
Pancreatic juice contains:
Cations: Na
+
, K
+
, Ca
2+
, Mg
2+
Anions: HCO
3

, Cl

, SO
4
2
, HPO
4
2
The digestive secretions of the pancreas are formed in the acini, rather similar to the acini of the salivary
glands and secreted into a network of ducts that meet to form the main pancreatic duct. This duct empties
into the duodenum. The pancreas secretes around 1 L of fluid per day; at rest this fluid is plasma-like, but it
becomes more alkaline at higher flow rates. This is because as it travels through the ducts, Cl

is exchanged for
HCO
3

ions. At higher flow rates there is less time for the exchange to take place. Pancreatic secretions neu-
tralize the acid chyme from the stomach in the duodenum to establish the optimum pH (67) at which pan-
creatic enzymes act.
Answers
46. See explanation
47. F T T F T
48. 1 E, 2 A, 3 B, 4 C, 5 F, 6 E
49. F T T F T
ONE STOP DOC 106
50. Concerning pancreatic enzymes
a. They are released in precursor form to prevent autodigestion
b. Pancreatic amylase differs in action to salivary amylase
c. Trypsinogen activates the proteases
d. Enterokinase is secreted via the main pancreatic duct
e. Pancreatic amylase is activated by Cl

ions
51. True or false. Cystic fibrosis
a. Affects 1 in 25 in white populations
b. Is characterized by absent or deficient pancreatic enzymes
c. Results in a deficiency in fat-soluble vitamins
d. Is caused by a defective sodium ion transporter
e. May affect the bronchi
CCK, cholecystokinin
Hepatobiliary system and pancreas 107
EXPLANATION: PANCREATIC SECRETION (ii)
Pancreatic secretion has a cephalic phase during which secretion is stimulated by vagal nerve activity.
In the gastric phase, the release of gastrin from the antrum stimulates secretion, but the major stimulus is in
the intestinal phase. Chyme entering the duodenum stimulates a bicarbonate- and enzyme-rich secretion of
fluid from the pancreas, as a result of the actions of secretin and CCK released from the mucosa of the small
intestine. Sympathetic stimulation inhibits pancreatic secretion.
THE ENZYMES The table below summarizes the pancreatic enzymes and their actions.
Digestive enzyme Activator Substrate Action
Trypsin(ogen) Enteropeptidase Proteins/polypeptides Cleaves peptide bonds adjacent to arginine
or lysine
Chymotrypsin(ogen) Trypsin Proteins/polypeptides Cleaves peptide bonds adjacent to arginine
or lysine
(Pro)elastase Trypsin Elastin Cleaves bonds next to aliphatic amino acids
Carboxypeptidase A and B Trypsin Proteins/polypeptides Cleaves C-terminal amino acids
Ribonuclease RNA Cleaves to release nucleotides
Deoxyribonuclease DNA Releases nucleotides
Co-lipase Trypsin Fat droplets Binds to make anchor for lipase
Pancreatic lipase Triglycerides Hydrolysis to free fatty acids and glycerol
Phospholipase A
2
Trypsin Phospholipids Release fatty acis and lysophospholipids
Pancreatic alpha-amylase Cl

Starch Hydrolysis of 14 linkages


Protease precursors (chymotrypsinogen, procarboxypeptidases) are activated by trypsin, which is formed
from trypsinogen by the action of enterokinase. Enterokinase is not secreted in the pancreatic duct but found
on the brush border of the small intestine mucosa. Pancreatic amylase splits alpha 1,4 linkages between starch
residues. Lipases are also present to act on triglycerides and phospholipids.
Cystic fibrosis affects 1 in 2500 white people. It is an autosomal recessive disorder characterized by recurrent
chest infections and exocrine pancreatic insufficiency. A defective chloride ion channel in the epithelial cells
causes decreased secretion of sodium and water. The result is the production of a particularly thick and viscous
mucus that cannot be easily cleared from the lungs and that blocks the pancreatic ducts. With time, the pan-
creatic acini atrophy. The median age of survival is 30 years.
Answers
50. T F F F T
51. F T T F T
ONE STOP DOC 108
52. Concerning the endocrine pancreas
a. Beta cells make up 75 per cent of the cells in the islets
b. Alpha cells secrete somatostatin
c. Islets of Langerhans are most plentiful in the head of the pancreas
d. Insulin is complexed with zinc in beta cell granules
e. Ca
2+
ion influx into the beta cell triggers insulin exocytosis
53. True or false? Glucagon
a. Is secreted by beta cells in the islets of Langerhans
b. Causes ketone body formation
c. Mediates its effects through a cAMP-dependent kinase
d. Is secreted in response to adrenaline
e. Is high in the plasma during the fed state
54. Hyperglycaemia
a. Is diagnosed at a fasting blood glucose of >7.8 mmol/L
b. Causes polyuria
c. Might be caused by an insulinoma
d. Increases sensitivity to insulin
e. May be managed through diet alone
cAMP, cyclic adenosine monophosphate; RBC, red blood cell
Hepatobiliary system and pancreas 109
EXPLANATION: INSULIN AND GLUCOSE CONTROL
The endocrine portion of the pancreas makes up just 2 per cent of the organ. Endocrine cells are localized
in islets of Langerhans.
Insulin is produced by beta cells, glucagon is produced by alpha cells and somatostatin is produced by delta
cells. Insulin is stored in vesicles in beta cells complexed with zinc. It has a biphasic pattern of secretion. No
matter what the stimulatory factor on the beta cell is, ultimately it is the intracellular rise in Ca
2+
ions that
causes insulin exocytosis.
Glucagon has antagonistic effects to insulin and is therefore a catabolic hormone. Stimulants of its release
from alpha cells are: falling blood glucose concentration, amino acids, catecholamines and gut hormones.
Glucogon binds to a G protein coupled receptor on its target cells, which stimulates the production of cAMP,
which in turn activates appropriate protein kinases to phosphorylate intracellular proteins. Deficiency of
glucagon does not normally cause a problem since other catabolic hormones can perform its function.
Hyperglycaemia is an excess of glucose in the blood. It is usually caused by insulin deficiency and/or insulin
resistance (diabetes mellitus). However, it can occur in rare cases of glucagonoma a tumour of the pancreas.
High blood glucose has a variety of detrimental effects on the body, outlined in the diagram opposite.
Glucose control in diabetes is generally managed through diet and insulin replacement therapy. Dietary
measures include switching from foods with readily assimilable sugars to slow-release carbohydrates, snacking
regularly (if taking insulin) and eating a high-fibre, low saturated fat diet
Answers
52. T F F T T
53. F T T T F
54. T T F F T
RBCs
Pancreatic
acini
Beta cell
Islet of
Langerhans
Alpha cell
Delta
cell
Cerebrovascular disease
Retinopathy
Ischaemic
heart disease
Nephropathy
Renal failure
Autonomic neuropathy:
Bladder dysfunction
Gastroparesis and diarrhoea
Postural hypotension
Peripheral neuropathy
Prone to infection
Polyphagia
Weight loss
Lack of energy
Glycosuria
Osmotic diuresis
Polyuria
Dehydration
Polydipsia Hypovolaemia
Short - term effects
Chronic effects
This page intentionally left blank
Index
abdominal aorta 46
abdominal pain 17, 67, 84, 85
abdominal wall
see also inguinal region
anterior 36, 37, 39, 77, 79
muscles 36, 37, 39
posterior 36, 37, 41, 43, 51
absorption
in the colon 62, 63
disorders of 57, 58, 59
in the small intestines 53, 54, 55,
56, 57
acetylcholine (ACh) 19, 31
achalasia 20, 21
acid reflux oesophagitis 15, 21,
29
acini
hepatic 75
pancreatic 105, 107
salivary 3
adenocarcinoma
gastric 29
pancreatic duct 99
adenomatous polyps 29
adenosine triphosphate (ATP) 87
adenylate cyclase 15
adipocytes (fat cells) 23, 87
adrenergic stimulation 3
adsorbent agents 65
adventitia 25
agar 65
alanine aminotransferase (ALT) 93,
95
albumin 75, 81, 103, 105
alcohol consumption
and liver function 93, 95, 97
and the pancreas 99
aldosterone 63, 105
alkaline phosphate enzyme 103
alkalinity 3, 53, 103, 105
allergies, food 59
alpha subunits 87
aluminium salts 15
amine precursor uptake and
decarboxylation (APUD) cells
27
amino acids 19, 33, 53, 83, 93
absorption 57
D-amino acids 55
deamination in the liver 75
feeding centre inhibition 23
and glucagon release 109
of insulin 91
L-amino acids 55
types 57
ammonia, free 13
amoxycillin 17
ampulla (duodenum) 43
ampulla (rectal) 69
ampulla of Vater 83
amylase
pancreatic 53, 107
plasma 99
salivary 3, 53
anaemia 59, 71
pernicious 29
anal canal 25, 50, 51, 68, 69
superior 51
anal column 69
anal sphincter 69, 71
anal valve 69
anastamoses 79, 97, 99
androgen steroids 103
angina, unstable 13
angular notch 26, 27
anions 105
anorectal canal 51
anorectal junction 69
anorexia 95, 99
antacids 15, 17
antibiotics 13, 17, 61, 85
anticholinergic agents 3
anticoagulants 15
antrum 13, 19, 31, 107
anus 68, 69, 71
aorta 21, 46
aphthous ulcers 105
appendices epiploicae 45
appendicitis 66, 67
appendix (vermiform) 43, 45, 46, 47,
49
ascites 97, 105
aspartate aminotransferase (AST) 93,
95, 103
aspirin 12, 13, 101
atropine 3
auditory tube (salpingopharynegeus) 7
Auerbachs (myenteric) plexus 21, 25,
31, 67
autodigestion 99
autoimmune conditions 29, 95, 97,
103
autonomic (extrinsic) nervous system
31, 33
autophosphorylation 87
bacteria 3, 19, 61, 81
barium meal radiology 21
Barretts oesophagus 28, 29
basolateral membrane 55
beta subunits 87
bicarbonate ions 53, 55, 63, 107
bile 19, 53, 75, 89, 99
composition 81
function 80
reabsorption 81
secretion 59, 80, 81
storage 83
bile acids 80, 81
bile canaliculi 78, 79
bile duct 79, 80, 83, 85, 99
common 89, 99
removal 99
bile pigments 81
see also bilirubin
bile salts, primary/secondary 81
biliary cirrhosis 97
biliary colic 85
biliary tract disease 99
bilirubin 61, 81, 103
in hepatitis 95
and jaundice 93, 100, 101
secretion 93
biliverdin 81
biopsy 29
bladder, epithelium 51
bleeding 99
due to peptic ulcers 13, 15, 17
of inflammatory bowel disease 67
rectal 70, 71
bleeding disorders 59
blood
oxygenated 75
vomiting 17
INDEX 112
bolus 7
bowel disease 59, 66, 67, 105
bowel intraluminal pressure 67
brain 87
bran 65
bronchus, left 21
bruising 105
Brunners glands 53
brush border 55, 107
brush border enzymes 53, 59
Budd-Chiari syndrome 97
C3-C5 nerves 21, 85
C-peptide 91
Ca
2+
ions 83
and gut motility 31
pancreatic 99, 105, 109
caecal bud 49
caecum 43, 47, 49
canal of Hering 79
cancer 21
see also tumour
gastric 28, 39
pancreatic 59, 98, 99
rectal 71
of the small intestine 59
carbohydrates 53
carbon dioxide 55
carbonic acid (HCO
3
) 55
carboxypeptidase A and B 107
cardiac sphincter 27, 45
carrier proteins 55
case studies
gallstones 84, 85
haemorrhoids 70
hiatus hernia 20, 21
inguinal hernias 40
pancreatitis 98, 99
peptic ulcer 12, 13
catecholamines 109
cations 105
caudal branchial arches 9
cefuroxime 61
Chagas disease 21
chalk 65
charcoal 65
chief cells 27
children, defecation 71
chlorpromazine 103
cholagogues 83
cholangitis 85
cholecystectomy 85
cholecystitis, acute 85
cholecystokinin (CCK) 19, 23, 33,
83, 107
cholestasis 93, 103
extrahepatic/intrahepatic 101
cholestatic (obstructive) jaundice 85,
99, 101, 103
cholesterol
absorption 57
of bile 81
deposition around eyelids 105
gallstones 85
synthesis in the liver 75
choleterics 81
cholinergic stimulation 3, 19, 33
chylomicrons 57
chyme 19, 33, 53, 63, 107
acidic 105
isosmotic to plasma 57
chymotrypsin/chymotrypsinogen 107
cimetidine 13
circular muscle 11, 27, 45, 67
cirrhosis
biliary 97
of the liver 95, 96, 97, 105
Cl

ions
absorption 55, 63
and bile 83
pancreatic 105, 107
salivary 3
secretion 53
cloaca 51
Clostridium difficile 61
clotting factors 75, 105
co-lipase 53, 107
coeliac disease 58, 59
coeliac plexus 75
coeliac trunk 27, 43, 45
coffee 99
colic arteries 45
coliforms 61
collagen 95, 97
colloids 13
colon 43, 44, 45
absorption 62, 63
ascending 43, 49, 51
and bile salts 81
connective tissue 67
descending 43, 45, 51
portal anastamoses 79
sigmoid 51, 69, 71
transverse 26, 43, 49, 51
coma 93
commensal bacteria 61
congenital abnormalities 9, 10, 11,
57, 67
conjugation 93, 101, 103
connective tissue 9, 67
constipation 64, 65, 71
constrictor muscles (pharynx) 7
contraction 31, 69
councilman bodies 94
covalent bonds 103
cremasteric artery/vein 39
Crohns disease 59, 67
crypts 25, 59
crypts of Lieberkhn 53
cyclic adenosine monophosphate
(cAMP) 15, 53, 109
cyclooxygenase (COX) 13
cystic artery 83
cystic duct 83, 85, 89
cystic fibrosis 59, 97, 106, 107
cystinuria 57
cytomegalovirus 95
cytosol 91
death 65, 67, 97
deep ring (inguinal canal) 39, 41
defecation 69, 70, 71
see also faeces
dehydration 63, 65
deoxyribonuclease 107
dermatomes 21
detoxification 75, 93
development 8, 9, 45
liver 88, 89
midgut 4851
pancreas 88, 89
stomach 10, 11
diabetes mellitus 91, 93, 99, 109
diaphragm 21, 27, 45, 75, 85, 89
Index 113
diarolyte 65
diarrhoea 61, 62, 63, 67
management of acute 64, 65
diet, and diabetes management 109
diffusion 55, 57
digestion 3, 19, 52, 53
digestive enzymes 55, 81
pancreatic 52, 53, 59, 99, 1057
digestive system, development 8, 9
disaccharides 53, 61
disulphide bonds 15, 87, 91
diverticular disease (diverticulosis) 66,
67
Downs syndrome 67
drowsiness 93
drug metabolism 100, 101, 102
conjugation 101, 103
first pass metabolism 101, 102
phase I 101
phase II 101, 103
drugs, hepatotoxic 95, 102, 103
ductus deferens 41
artery of 39
duodenal papilla 83
duodenal peptic ulcers 13
duodenal stenosis/atresia 49
duodenojejunal junction 43
duodenum 45, 52, 99
absorption 57
anatomy 42, 43
bile secretion 83
descending 43, 83
development 48, 49, 51
distension/flow into 27, 33
and gastric secretions 19
and pancreatic secretions 105, 107
removal 99
dysphagia 21
ectoderm 9, 51
elastase 107
electrolyte replacement 65
embryo 9
embryonic disc 9
empyema 85
encephalopathy, hepatic 93, 105
endocrine glands 75, 91, 99, 108,
109
see also G-cells (endocrine cells)
endocrine insufficiency 99
endoderm 9, 89
endopeptidases 53
endoplasmic reticulum (ER), rough
91
endoscopy 21, 29
endothelium, hepatic 78
energy expenditure 23
enteric (intrinsic) nervous system 30,
31, 33
myenteric (Auerbachs) plexus 21,
25, 31, 67
submucous (Meissners) plexus 25,
31, 67
enterocolitis, ischaemic 67
enterocytes 53, 55
enterohepatic recirculation 81
enterokinase 107
enzymes 93
see also digestive enzymes
epigastric arteries
inferior 37, 39
superior 37
epigastric veins, inferior/superior 37
epiglottis 5, 7
epiploic foramen 11
epithelium 13, 56, 61
bladder 51
development 9
endodermal 89
glandular 29
small intestine 49, 53
squamous 29
villi 55
Epstein-Barr virus (EBV) 95
erythrocytes 81, 87
Escherichia coli 85
exocrine glands 99, 104, 1057
exocrine insufficiency 99
exocytosis 91
exophthalmia 105
external oblique muscle 37, 39
faecal-oral transmission route 95
faeces 61, 63, 69, 71, 81
see also defecation
falciform ligamen 77
fascia 39
fasting, overnight 91
fat
digestion 53, 81, 87
high fat diets 99
metabolism 23
fatty acids 19
absorption 57
and cholecystokinin secretion 83
free 53, 99
in the liver 75
short-chain 61
synthesis 87
fatty change, of the liver 92, 93
feeding centre 22
femoral nerve 37
fetus 8, 9, 79, 89
fever (pyrexia) 84, 85, 95
fibre 61, 65, 67
fibrosis
gallbladder wall 85
hepatic 93, 95
finger nails, clubbing 105
fixation, of the intestines 51
flapping tremor 105
flora of the gut 29, 60, 61
fluid therapy 13, 63, 65, 85
folate deficiency 59
food allergies 59
foregut 9, 45, 89
free radicals 103
fructose 55
G protein 109
G-cells (endocrine cells) 19, 27, 33
galactose 55
gallbladder 82, 83
anatomy 83
contraction 83
development 89
perforation 85
removal 99
gallbladder fossa 77
gallstone ileus 85
gallstones 84, 85
Ca
2+
bilirubinate 85
cholesterol 85
mixed 85
INDEX 114
gamma glutamyl transferase (GGT)
103
ganglion cells 25, 67
gastric acid secretion 13, 14, 15, 18,
27, 33
duodenal protection from 53
and gastric sterility 61
inhibition 14
phases of 19
gastric arteries 27
gastric cancer 28, 39
gastric emptying 23, 33
gastric glands 27
gastric lipase 53
gastric mucosa 13, 15, 25
and gastritis 29
healing following ulceration 17
rugae 27
zones 26
gastric pits 27
gastric submucosa 29
gastrin 19, 27, 33, 107
gastritis 13, 15, 28, 29
gastroduodenal artery 27
gastroepiloic artery 27
gastrointestinal inhibitory peptide
(GIP) 19
genioglossus muscle 5
genitofemoral nerve 39, 41
germ cells 9
gliadin 59
globlet cells 53
glomerular filtration rate (GFR) 93
glucagon 108, 109
glucagonoma 109
gluconeogenesis 87, 105
glucose 105, 109
control 91, 108, 109
digestion 53
feeding centre inhibition 23
hyperglycaemia 99, 108, 109
synthesis in the liver 75
transport 55
glucose tolerance, impaired 90, 91,
99
GLUT-4 transporters 87
glutathione 103
gluten 59
glycerol 53
glycogen 75, 87, 91, 105
glycolysis 87
glycoproteins 3, 13
goblet cells, mucus-secreting 25
gram-negative bacteria 13, 61
gram-positive bacteria 61
groin, hernia 40, 41
growth 91
gut motility 30, 31, 32, 33, 65
slow/spike waves 31
gut tube, primitive 9
gynaecomastia 105
H
+
/K
+
ATPase (proton pump) 15
H
2
antagonists 13, 17
habit conditioning 23
haematopoiesis 79, 89
haemoglobin 81
haemorrhoids 70, 71
hair loss 105
halothane 103
hard palate 7
Hartnup disease 57
haustra (large intestine) 45
HCl 63
HCO
3

ions 3, 105
Helicobacter pylori 12, 13, 17, 29, 61
hepatic arteries 27, 79
common 83
right 83
hepatic diverticulum 89
hepatic duct, common 83
hepatic encephalopathy 93, 105
hepatic plexus 75
hepatic portal vein 78, 79
hepatic vein, occlusion 97
hepatitis 94, 95, 97, 103
acute 94, 95
hepatocytes 75, 78
bile secretion 81, 89
insulin effects 87
and liver failure 93
necrosis 103
hepatorenal syndrome 93
hepatotoxic drugs 95, 102, 103
hernia
hiatus 20, 21
inguinal 39, 40, 41
midgut 49
mucosa/submucosa 67
hernial sac, excision 41
hexoses 55
hiatus hernia 20, 21
paraoesophageal 21
sliding 21
hindgut 9, 45, 51
Hirschsprungs disease 66, 67
histamine 19
histamine receptor antagonists 15, 17
hormones 105, 109
HPO
4
2
ions, pancreatic 105
hydrochloric acid 19
hydrogen ions 15, 55, 63
hydrolysis 101
hyperglycaemia 99, 108, 109
hyperlipidaemia 99
hyperphagia 23
hypertension, portal 96, 97
hyperthyroidism 105
hypertrophy 11
hypoalbuminaemia 105
hypoglossal nerve 5
hypothalamus 22, 23
hypovolaemic shock 13, 15, 99
hypoxia 93
ileocaecal artery 47
ileocaecal valve 43
ileum 43, 57, 81
iliacus 37
iliogastric reflex 33
ilioinguinal nerve 39, 41
immune response 59
immunoglobulins (Igs) 3
inflammatory bowel disease 59, 66,
67, 105
inflammatory liver disease 95
inguinal canal 38, 39, 41
inguinal hernias 39, 40, 41
direct/indirect 41
inguinal ligament 38, 39
inguinal nodes 39
insulin 90, 91
accumulation in the blood stream
105
Index 115
deficiency 99, 109
effects of 86, 87, 91
and glucose control 91, 108, 109
secretion 89, 91, 109
storage 109
insulin receptor 86, 87
insulin replacement therapy 109
insulin resistance 109
intermediates 103
internal oblique muscle 37, 39
intra-abdominal pressure 41
intrinsic factor 19
ions, absorption 55
iron deficiency 59
IRS 87
ischaemic enterocolitis 67
islets of Langerhans (endocrine) 99,
108, 109
alpha cells 109
beta cells 91, 109
delta cells 109
Ito cells 95
jaundice 93, 95, 97, 100, 101
cholestatic/obstructive 85, 99, 101,
103
prehepatic 101
jejunal mucosa 59
jejunum 42, 43, 57, 99
K
+
ions
of bile salts 81
intestinal 55, 63
pancreatic 105
salivary 3
kidney 26, 93
Klebsiella 85
L1-L4 nerves 37, 45, 99
lactase 53
lacteal 55
lactulose 65
lacunar ligament 39
lamina propria 57
laparoscopy 41
large intestine 44, 45
see also appendix (veriform);
caecum; colon
anatomy 42, 43
blood supply 43, 45
mechanical efficiency 65
mucosa 25
laryngotracheal tube 9
larynx 7
laxatives 64, 65
bulk/osmotic 65
leptin 23
lesser sac 11
lethargy 93
levator ani (pelvic diaphragm)
muscles 69
levator veli palatine 5
lifestyle advice, for peptic ulcers 13
lignocaine 101
lingual frenulum 5
lingual septum 5
lipase 87, 107
gastric 53
lingual 3
pancreatic 53, 81, 99
salivary 3
lipid peroxidation 103
lipids, bile 81
lipogenesis 87, 91
lipoproteins 75
liver 49
see also hepatic...; hepatitis;
hepatocytes
anatomy 76, 77
bile secretion 80, 81
blood supply 79, 97
development 9, 88, 89
drug metabolism 100, 101, 102
fatty changes to 92, 93
function 74, 75
hepatotoxic drugs and 102,
103
innervation 75
jaundice 101
lobes 75, 76
microstructure 78, 79
necrosis 93, 95, 101
palpability 76
pathology 94, 95, 102
regeneration 95
structure 74, 75
liver bud 89
liver cirrhosis 95, 96, 105
macronodular/micronodular 97
liver disease 93, 104, 105
liver failure 92, 93, 97, 103
acute 92, 93
consequences 104, 105
liver function tests 103
liver parenchyma 89
lobules 75
loperamide 65
lubrication, salivary 3
lumbar plexus 37
lumbrosacral trunk 37
lymph 39
lymph nodules 53
lymphatic system 37, 39, 57
see also spleen
lymphocytes 57, 59
lymphoid tissue, aggregations/diffuse
distribution 57
lysozyme 3
McBurneys point 47
magnesium aluminium silicate 65
magnesium ions 105
magnesium salts 15, 65
malabsorption 58, 59
malaise 95
maltase 53
mandible 5
masseter 5
mast-like cells 19
mastication 4, 5, 7, 23
mechanoreceptors, gastric 19, 23
Meckels diverticulum (ileal
diverticulum) 51
mediastinum, posterior/superior
21
Meissners (submucous) plexus 25,
31, 67
mesenchyme 9, 49
mesenteric arteries
inferior 43, 45, 51, 69
superior 43, 45, 47, 49
mesenteric veins
inferior 71, 79
superior 79
INDEX 116
mesentery 9, 11
anterior 77
of the appendix 47
of the colon 51
dorsal 9, 11
of the intestines 43, 51
of the liver 77
of the midgut 49
ventral 9, 89
mesoappendix 47
mesoderm 9
metabolic acidosis 63
metabolism 75, 93
drug 100, 101, 102, 103
metaplasia, intestinal 29
methycellulose 65
metoclopramide 65
metronidazole 17, 61
micelles 53, 57
microvilli 55
midgut 9, 45, 4851
midgut herniation 49
caudal/cranial part 49
midgut loop 49
monochloramine 13
monoglycerides 57
monosaccharides 53
morphine 101
motor neurons 31
mouth 3, 5
mucins 61
mucosa 25
absorptive 25
epithelium 25
gallbladder 83
gastric 13, 15, 17, 25, 26, 27, 29
herniation 67
intestinal 53, 55, 57, 59, 63, 81,
107
lamina propria 25
muscularis mucosa 25, 27
protective 25
secretory 25
sublayers 25
mucosal defences, impaired 13, 14,
15
mucosal-associated lymphoid tissue
(MALT) 57
mucus
of cystic fibrosis 107
intestinal 53, 70, 71
mucus-secreting cells 25, 27
Murphys sign 85
muscarinic receptor antagonists 17
muscle
see also smooth muscle
abdominal wall 36, 37, 39
circular 11, 27, 45, 67
development 9
insulin effects 87
involuntary 21, 69
of mastication 4, 5
oesophageal 9, 21
pharyngeal 7
rectal 69
of the soft palate 5
striated 9
of the tongue 5
voluntary 21, 69
muscular tube, gastrointestinal tract
as 25
muscularis, circular/longitudinal 25
muscularis mucosa 25, 27
muscularis uvulae 5
myeloperoxidase 13
myenteric (Auerbachs) plexus 21, 25,
31, 67
myocardial infarction (MI) 13
myoepithelial cells 3
N-acetyl-p-benzoquinoneimine 103
Na
+
ions
absorption 55, 63
and active transport 55
bile 81, 83
pancreatic 105, 107
salivary 3
in the urine 93
Na
+
/K
+
ATPase pump 55
NaHCO
3
63
nasopharynx 7
nausea 17, 95
see also vomiting
neck, pain 21
neck cells (gastric) 27
necrosis
hepatic 93, 95, 101
pancreatic 99
neisseria 61
neuropeptide Y 23
neutralization 105
neutrophils 13
nitrogenous compounds 93
nitrosamines 29
nodules, of liver cirrhosis 97
non steroidal anti-inflammatory
drugs (NSAIDs) 13, 15, 29
see also aspirin
non-covalent bonds 103
noradrenaline 3, 31, 33
nutrients, absorption 55, 56
obstruction
intestinal 85
of the lower oesophageal sphincter
21
obstructive jaundice 85, 99, 101, 103
obturator nerve 37
oedema 59, 99, 105
oesophageal atresia 9
oesophageal sphincter
lower 21, 33
upper 7
oesophageal stricture, lower 29
oesophagus 7, 20, 21, 26, 45
development 8, 9
disorders of 9, 29
mucosa 25
portal anastamoses 79
zones 21
oestrogen 105
omental bursa 11
omentum
greater 11
lesser 77
omeprazole 13, 15, 17
oncotic pressure 105
oral cavity 4, 5, 25
oropharynx 5, 7
osmosis 55
osmotic diarrhoea 59
osmotic gradients 57, 63
osmotic laxatives 65
oxidation 101
Index 117
pacemaker, of gut smooth muscle 31
pain
abdominal 17, 67, 84, 85
of pancreatic cancer 99
of peptic ulcers 13
referred 21, 85
stomach 12, 13
umbilical region 67
pain relief 17, 85
palate 5
palatoglossal arches 5
palatoglossus 5
palatopharangeus 5
palpitation, abdominal 76, 84
pampiniform plexus 39
pancreas 26
development 9, 88, 89
endocrine portion 91, 99, 108,
109
exocrine portion 99, 104, 1057
removal 99
pancreatic bud, dorsal/ventral 89
pancreatic cancer 59, 98, 99
pancreatic duct 83, 89, 99, 107
accessory 89
main 89, 105
pancreatic enzymes 52, 53, 59, 99,
1057
pancreatic insufficiency 59
pancreatic secretion 19, 523, 59, 81,
99, 1047
phases of 107
stimulation 33
pancreatic surgery 59
pancreatic tumour 109
pancreaticoduodenal artery,
inferior/superior 45
pancreatitis 85
acute 98, 99
chronic 59
para-aortic nodes 39
paracetamol 95, 103
parasympathetic activity 3, 25, 33, 71
parenchyma 9
parenteral transmission route 95
parietal (oxyntic) cells 13, 15, 19, 27
H
2
receptors 15
pectinate line 69
pentoses 55
pepsin 19, 53
pepsinogens 19, 27
peptic ulcers 12, 13, 15, 21, 29
symptoms 16, 17
treatment 16, 17
peptides 19, 53
perianal complications 71
pericardium 21
pericholecystic fluid 84
perineal tendon/body 51
peristalsis 31, 33
absent 21
of the bile duct 83
and defecation 71
disorders of 67
as reflex response 33
peritoneal cavity 11, 105
peritoneal coverings 10
peritoneum 47, 67, 69, 85
parietal 21, 51
pernicious anaemia 29
Peyers patches 57
pH
gastric influences on 15, 19
pancreatic influence on 105
small intestinal influences on 53
pharynx 6, 7, 25
phospholipases 13
phospholipase A
2
107
phospholipids 107
phosphorous toxicity 95
phosphorylation/dephosphorylation
cascade 87
phrenic nerve 21, 75
plasma 57
plasma amylase 99
plasma proteins 75, 105
pleura 21
polypeptide hormones 91
porta hepatis 79
portal hypertension 71, 96, 97
portal triad 75, 79
portal vein 57, 78, 81, 97
portosystemic anastamosis 97
pregnancy 70, 71, 93
preproinsulin 91
primitive gut 9, 49
processus vaginalis, developmental
41
proctodeum membrane 51, 69
proelastase 107
proinsulin 91
propanolol 101
prostaglandins 13
prostaglandin 2 (PGE
2
) 13
protease 13, 107
protein 59
absorption 55
digestion 19, 53
high protein diets 99
hydrolysis 53
synthesis 87, 91
protein kinases 15, 109
prothrombin time 103
proton pump inhibitors 13, 15, 17
proton pumps 15, 19
pruritus ani 71
psoas major 37, 67
pterygoid, lateral/medial 5
pubic tubercle 39
pubo-rectalis 69
pudendal arteries, internal 51
pudendal nerves 69, 71
pyloric sphincter 27, 33, 45
pyloric stenosis 10, 11
pylorus 11, 26, 31, 45
quadratus lumborum 37
R protein 3
ranitidine 15
rectal ampulla 69
rectal arteries
inferior 51, 69
middle 69
superior 45, 51, 69
rectal carcinoma 71
rectal veins, superior 71
rectum 45, 68, 69
development 51
distension 71
portal anastamoses 79
rectus abdominis muscle 36, 37
rectus sheath 37
reduction 101
INDEX 118
reflexes
cholinergic response 19
of defecation 71
peristalsis as 33
swallowing 7
vagal 33
reflux gastritis 29
reflux oesophagitis 15, 21, 29
regurgitation, prevention 27
rehydration solutions, oral 63, 65
resuscitation 13
reticuloendothelial system 81
retrocaecal structures 47
retroperitoneal structures 10, 26, 43,
45, 51
Reyes syndrome 93
rib 76
ribonuclease 3, 107
right iliac fossa 43, 67
rugae (gastric mucosa) 27
S2/S3 vertebrae 69
sacral flexure 69
sacral nerves 2-4 71
saline, intravenous 13
saliva 2, 3, 7
composition 3
functions 3
water brash (sudden salivation)
21
salivary amylase 3, 53
salivary ducts, excretory/intercalated
3
salivary glands 2, 3
salivons 2, 3
satiety 22, 23
satiety centre 23
scrotum 39, 41
secretin 19, 33, 53, 81, 107
senna 65
septum transversum 89
serine residues 13, 87
sexual characteristics, loss of
secondary 105
shock
hypovolaemic 13, 15, 99
systemic 93
shoulder, referred pain 21, 85
sinusoids 78, 97
skeletal muscle 69
sleep disturbances 17
small intestine 44, 49, 56
see also duodenum; ileum; jejunum
absorption 54, 55, 57
anatomy 42, 43
bile secretion 81
blood supply 43
cancer 59
fixation 51
function 52, 53
infarction 59
malabsorption 59
mucosa 25
resection 59
secretions 52, 53
smell, sense of 23
smoking 67, 85, 99
smooth muscle 30, 31, 71
circular 27
of the internal anal sphincter 69
myogenic nature 31
of the oesophagus 9
phasic contraction 31
SO
4
2
ions, pancreatic 105
soft palate (palatopharyngeus) 5, 7
somatostatin 109
space of Disse 78, 79, 95
speech 3, 7
spermatic cord 38, 39
constriction 41
sphincter of Oddi 83
sphincters
see also specific sphincters
tonic (continuous) contraction 31,
69
spider naevi 105
spine
see also vertebral column
anterior superior iliac 39, 47
spleen 26, 46, 47
rupture 47
splenectomy 47
splenic artery 27, 47
splenic vein 79
splenomegaly 97
spotty necrosis 95
starch 3, 107
starvation 93
steatorrhoea 59, 99
stercobilinogen 61
steroids 15, 103
stomach 10, 11
see also gastric...; gastritis
antrum 13, 19, 31, 107
blood supply 26, 27
cardiac sphincter 45
development 8, 9, 10
disorders 28, 29
distension 19, 23
fat digestion 53
flora of the gut 61
fundus 19, 26, 53
mechanoreceptors 23
motility 33
pacemaker region 31
physiology of 26, 27
protein digestion 53
pyloric sphincter 45
removal of distal 99
rotation 90
o
clockwise 11
sterile environment 61
ulcers 12, 13
stools
floating 59
mucus 70
pale 101
Streptococcus faecalis 85
striated muscle 9
stroke 13
styloid process (stylopharangeus) 7
submucosa 25
herniation 67
sucrase 53
superficial ring (inguinal canal) 39
suprarenal gland 26
surgery
for congenital pyloric stenosis
11
and gastritis 29
for hitatus hernia 21
for inguinal hernia 41
intestinal 61
pancreatic 59, 99
sutures 41
Index 119
swallowing 3, 6, 7
difficulties 21
and satiety 23
voluntary/involuntary phases 7
sympathetic activity 3, 33, 107
T7-T12 nerves 21, 37, 45, 67
taeniae coli 45
taste, sense of 23
temporalis 5
tensor veli palatini 5
testicular artery/vein 39
testis 39
thigh flexor 37, 67
threonine residues 87
tight junctions 63
tongue 3, 4, 5, 7
tonsils 5, 57
trachea 21
tracheoesphageal septum 9
transport mechanisms 55
transversus abdominis muscle 37
triacylglycerol 87
triglycerides 3, 53, 57, 75, 93, 107
trilaminar embryonic disc 9
triple therapy 13, 17
truss 41
Trypanosoma cruzi 21
trypsin 53, 107
trypsinogen 107
tubular glands 25
tumour
pancreatic 109
perianal 71
tyrosine kinase domain 87
tyrosine kinase receptors 87
ulcerative colitis 67
ulcers
aphthous 105
peptic 1213, 15, 1617, 21,
29
umbilical cord 49
umbilical region 67
umbilical vein 89
umbilicus 47
urea 75
breath test 13
urease 13
urethra 51
urine 93, 101
urobilinogen 61
urogenital sinus 51
urorectal septum 51
vagal (iliogastric) reflex 33
vagus nerve 19, 75
and bile secretion 81
and gallbladder contraction 83
and gut motility 33
and pancreatic secretion 107
and satiety 23
valvulae conniventes 55
vas deferens 39
vascular cushions 71
vena cava, inferior 77
vertebral column 21, 69
see also spine
vesicles, for insulin storage 109
vestibule 5
villi, intestinal 25, 53, 54, 55, 59
viruses 3, 95
vitamin A 75
vitamin B12 59, 75
vitamin D 75
vitamin K 59
vitamins 61
vitelline vein 89
vomiting 11, 12, 13, 17, 84
see also nausea
water 65, 107
absorption 25, 55, 57, 63
of bile 81, 83
water brash (sudden salivation) 21
water retention 105
weight control 22
weight loss 59, 67, 99
wheat, allergies to 59
Whipples procedure 99
xanthelasmata 105
yolk sac 9, 49
yolk stalk 49, 51
zinc 109
zonal necrosis 95
zymogen granules 3

You might also like